You are on page 1of 108

curso de post-grado para profesores

especialidad en matematica
GEOMETRIA EUCLIDEANA
Equipo de Diseno:
Nahomy Jhopselyn Hernandez Cruz
Gabriel Alexander Chicas Reyes
Eduardo Arnoldo Aguilar Canas
Hector Enmanuel Alberti Arroyo
Ernesto Americo Hidalgo Castellanos
Juan Agustn Cuadra
Claudia Patricia Corcio Lopez de Beltran
Carlos Mauricio Canjura Linares
Oscar Armando Hernandez Morales
Aaron Ernesto Ramrez Flores
5 de abril de 2010

Indice
1. Angulos entre paralelas. 2

2. Triangulos: Teoremas Fundamentales. 7

3. Congruencia de Triangulos. 16

4. Cuadrilateros: Clasificacion y Propiedades. 24

5. Angulos en la Circunferencia. 30

6. Teorema de Thales y su recproco. Semejanza de Triangulos. 46

7. Puntos y Rectas Notables del Triangulo. 58

8. Solucion a Problemas Selectos. 72

1
1. Angulos entre paralelas.
ANGULOS
Definimos como angulo a la figura geometrica formada por dos rayos (o semirrectas) distintas
que tienen el mismo origen. Ese origen se llama vertice del angulo. Al angulo de vertice O y
rayos OA y OB se le denota AOB.

Dos angulos AOB y BOC son adyacentes si y solo si tienen un lado comun OB y los lados
no comunes OA y OC estan en semiplanos distintos, determinados por el lado comun.

Bisectriz de un angulo es la semirrecta que lo divide en dos angulos adyacentes iguales.

Dos angulos son:

Congruentes o Iguales: si tienen igual medida.

Suplementarios: si su suma es 180.

Complementarios: si su suma es 90.

Por otra parte, dos rectas en el plano pueden ser secantes o paralelas,1 dependiendo si se cortan
o no; ademas, si las rectas son secantes, el punto de corte es unico, y definen cuatro angulos,
que se agrupan por parejas en angulos opuestos por el vertice (las parejas de angulos tales que
uno esta formado por la prolongacion de los lados del otro).

Los angulos opuestos por el vertice son iguales (Justifique), por lo que dos rectas secantes for-
man cuatro angulos que definen dos parejas de angulos iguales, y si tomamos un miembro de
cada pareja, se tienen dos angulos suplementarios. En particular, si las rectas son secantes y
forman cuatro angulos iguales, seran llamadas rectas perpendiculares,2 y los angulos as gene-
rados son llamados angulos rectos. Y como es muy conocido, un angulo agudo es aquel cuya
medida es menor a la de un angulo recto, y un angulo obtuso es aquel cuya medida es mayor
que un angulo recto; en particular, un angulo obtuso sera llamado angulo llano si su medida es
el doble que la de un angulo recto.

ANGULOS ENTRE PARALELAS


Al intersecar un par de rectas paralelas por una recta llamada transversal o secante, se forman
los siguientes tipos de angulo:

Angulos Correspondientes: Son dos angulos no adyacentes situados en el mismo lado de


la secante, uno en el interior y otro en el exterior de las paralelas.

Angulos Alternos Internos: Son dos angulos no adyacentes situados en el interior de las
paralelas, y en distintos lado de la secante.
1
Si la recta AB es paralela a la recta CD, se denota AB k CD.
2
Si la recta AB es perpendicular a la recta CD, se denota AB CD.

2
Angulos Alternos Externos: Son dos angulos no adyacentes situados en el exterior de las
paralelas, y en distintos lado de la secante.
Angulos Conjugados: Son los angulos no adyacentes situados uno en el interior y el otro
en el exterior de las rectas paralelas y del mismo lado de la secante.
Las propiedades fundamentales de los angulos entre paralelas son:
1. Los angulos correspondientes son iguales entre s.
2. Los angulos alternos internos son iguales entre s.
3. Los angulos alternos externos son iguales entre s.
4. Los angulos conjugados son suplementarios.

Figura 1: Angulos entre las rectas paralelas L1 y L2 .

Ejercicios
1. Tres angulos adyacentes forman un semiplano y tienen sus medidas proporcionales a los
numeros 5, 7 y 8. Hallar la medida del menor angulo.
2. Demostrar que las bisectrices de dos angulos suplementarios son perpendiculares.

3. En la figura adjunta, L1 k L2 y
L3 k L4 . Calcular x.

3
4. Con ayuda de la figura 2, demuestre que: Si L1 k L2 entonces = + .

Figura 2

5. En la figura 3, AB k F G. Hallar el angulo x si el AM F = 90 y el M AB = 110.

Figura 3

6. Calcular el OP Q, si OP es bisectriz del angulo O, L1 k L2 y P Q L1 . Ver figura 4.

Figura 4

4
7. En la figura 5, L1 k L2 y L3 k L4 , calcular .

Figura 5

8. En la figura 6, calcular x, si L1 k L2 .

Figura 6

9. Calcular la medida del grafico anexo, si las


rectas L1 y L2 son paralelas.

5
10. En la figura 7, L1 k L2 y L3 k L4 . Hallar el valor del angulo .

Figura 7

11. Sea AOB = 24, en la region exterior a dicho angulo se traza el rayo OC. Hallar la
medida del angulo formado por las bisectrices de los angulos AOC y BOC.

12. Del grafico 8, calcular y, cuando x tome su maximo valor entero.

Figura 8

6
2. Triangulos: Teoremas Fundamentales.
TEOREMAS FUNDAMENTALES EN TODO TRIANGULO.
Diremos que tres puntos que pertenecen a una misma recta son puntos colineales; de manera
analoga, si tres rectas pasan por un mismo punto, seran llamadas rectas concurrentes. Si toma-
mos al azar tres puntos en el plano, en muy raras ocasiones estos puntos estaran alineados,3
y diremos entonces que son los vertices de un triangulo; analogamente sucede con las rectas,
tres rectas por lo general no concurren, y la figura geometrica que estas definen es tambien un
triangulo.4 Una definicion completa para nuestros intereses es la siguiente:

Definicion de Triangulo. Si A, B y C son tres puntos cualesquiera no colineales (Ver figura


9), entonces la reunion se los segmentos AB, BC y AC se llama triangulo ABC y se denota
por 4ABC. Los puntos A, B y C se llaman vertices y los segmentos AB, BC y AC se llaman
lados. Simbolicamente: 4ABC = AB BC AC. Todo triangulo ABC determina tres angulos
internos o interiores: ABC, ACB y BAC, y se llamara angulo externo o exterior, al
angulo determinado por un lado y la prolongacion del lado adyacente, en la figura 9, , y
son angulos exteriores.

Figura 9: Elementos del Triangulo

Dado el 4ABC, se tiene que AB + BC + CA = p = 2s, donde p es llamado el permetro y s


el semipermetro del triangulo. Para abreviar, suele asociarse a cada vertice un lado opuesto,
y viceversa, por ejemplo, el lado opuesto de A es BC, y es frecuente que se denote por a;
analogamente b = CA, c = AB.

Teorema 1: En todo triangulo, la medida de un angulo exterior es igual a la suma de las


medidas de dos angulos interiores del triangulo no adyacentes a el.

La demostracion de este teorema se basa en las relaciones de angulos entre paralelas; se deja
al lector que haga la demostracion (Sugerencia: por un vertice, trace una recta paralela al lado
3
En teora de probabilidades, la probabilidad que esto ocurra es cero!
4
El termino mas riguroso para esta figura es trilatero. En este caso, habra que hacer una consideracion: si
hay un par de rectas paralelas, el trilatero definido ya no es normal segun nuestro sentido comun, sin embargo,
sigue siendo un trilatero!

7
opuesto)

Corolario: En todo triangulo, la suma de las medidas de sus tres angulos internos es igual a
180.

Teorema 2: Desigualdad Triangular. En todo triangulo, la longitud de uno de sus lados


esta comprendido entre la suma y la diferencia de los otros dos.

Sin ser muy rigurosos, suponga que dado el segmento AB se traza con centro en A una cir-
cunferencia de radio r1 , y con centro en B una circunferencia de radio r2 ; si AB < r1 + r2 ,
las circunferencias se cortaran en dos puntos, y cualquiera de ellos puede ser el vertice C,
as AB < BC + CA; en cambio, si AB = r1 + r2 o peor aun, si AB > r1 + r2 , la construccion
del 4ABC no es posible.

La Desigualdad Triangular es un resultado fundamental, a partir de esta y de su modelo de


demostracion se generan los Criterios de Congruencia de Triangulos; a groso modo, si dadas
ciertas condiciones, la construccion de una figura geometrica (un triangulo en particular) queda
determinada de manera unica, entonces dos figuras que reunen las mismas condiciones seran
llamadas figuras congruentes.

As, si se tienen tres segmentos (cuyas longitudes cumplen la desigualdad triangular), dejando
uno fijo y construyendo las circunferencias con centros en los extremos de este segmento y
radios las longitudes de los otros segmentos, por construccion, solo sera posible obtener dos
triangulos (uno con cada punto de interseccion de las circunferencias), que son basicamente el
mismo pero la orientacion de los angulos es contraria; as, si se sabe que dos triangulos cumplen
tener lados respectivamente iguales, por construccion, deben de ser iguales. Este es el conocido
criterio LLL de congruencia de triangulos; mas adelante se detallaran el resto de criterios, pero
a partir de este probaremos el siguiente resultado:

Teorema 3: En todo triangulo, se cumple que a lados iguales se oponen angulos iguales, y
viceversa.

Suponga que 4ABC es tal que AB = AC, entonces, por criterio LLL, 4ABC es congruente
al 4ACB (en ese orden, porque AB = AC, BC = CB y CA = BA), entonces, los angulos
que se oponen a los angulos iguales son iguales. Para el recproco necesitamos otro criterio de
congruencia, por lo que la demostracion se dejara incompleta; retome esto en la seccion de
congruencia de triangulos.

Teorema 4: En todo triangulo se cumple que a mayor lado se opone mayor angulo y viceversa.

Este teorema se deja como ejercicio para el lector (Sugerencia: utilice el teorema anterior, tome
el lado mayor y defina un punto adecuado que genere un triangulo con dos lados iguales.)

CLASIFICACION DE TRIANGULOS.
1. Con relacion a sus lados:

8
a) Escaleno: si sus tres lados no son congruentes.
b) Isosceles: si por lo menos dos de sus lados son congruentes.
c) Equilatero: si sus tres lados son congruentes (note un triangulo equilatero es tambien
isosceles, y que los tres angulos internos son iguales entre s e iguales a 60)

2. Con relacion a sus angulos internos:

a) Acutangulo: si su angulo mayor es agudo (note que entonces los tres angulos son
agudos)
b) Rectangulo: si su angulo mayor es angulo recto (note que el angulo en cuestion es
unico y que los otros dos angulos son agudos; as, en un triangulo rectangulo, la
hipotenusa es mayor a los catetos)
c) Obtusangulo, si el angulo mayor es angulo obtuso (note que el angulo en cuestion
es unico y que los otros son agudos; as, en un triangulo obtusangulo, el lado que se
opone al angulo obtuso es el lado mayor)

LINEAS NOTABLES EN UN TRIANGULO.


1. Altura: Se llama altura de un triangulo al segmento que parte de uno de sus vertices y
llega en forma perpendicular al lado opuesto o a su prolongacion.

2. Mediana: Se llama Mediana al segmento que une un vertice con el punto medio del lado
opuesto.

3. Mediatriz: Se denomina mediatriz de un lado de un triangulo es la recta perpendicular a


dicho lado en su punto medio.

4. Una Bisectriz: La bisectriz es la recta que divide en dos angulos iguales a un angulo
dado; en particular, es bisectriz interna si es la bisectriz de un angulo interno de un
triangulo, y bisectriz externa si es la bisectriz de un angulo externo de un triangulo.

DISTANCIA DE UN PUNTO A UNA RECTA.


En la figura 10, sea P un punto exterior a una recta L, la longitud de la perpendicular P M a
la recta L es la distancia del punto P a dicha recta. Esta perpendicular tiene la propiedad de
ser unica y su longitud es la distancia mnima del punto a la recta (Pruebelo utilizano el hecho
que la hipotenusa es mayor que los catetos).
Los segmentos P A y P B no son perpendiculares a L y se llaman oblicuas.

TEOREMA DE PITAGORAS.
Abordamos el estudio de las Relaciones Metricas, del cual solo realizaremos el analisis del fa-
moso Teorema de Pitagoras, cuyo enunciado es el siguiente:

Teorema: Pitagoras. En un triangulo rectangulo, el cuadrado de la hipotenusa es igual a la


suma de los cuadrados de los catetos.

9
Figura 10

Una demostracion de este teorema es debida a Thabit ibn Qurra (836-901), la cual consiste en
diseccionar la figura que se forma al construir dos cuadrados de lados respectivamente iguales
a los catetos de un triangulo rectangulo, como se muestra en el grafico 11.

Figura 11

Recproco del teorema de Pitagoras: Si en un triangulo el cuadrado de un lado es igual a


la suma de los cuadrados de los otros dos lados, el triangulo es rectangulo.5

Ejercicios.

1. En la figura adjunta ambos triangulos son


equilateros. Encuentre el valor de .

5
Ver demostracion en la seccion de congruencia de triangulos.

10
2. En la figura 12, calcular el x si el AOB = 100 y L1 k L2 .

Figura 12

3. (*) En la figura 13, ABDE es un cuadrado y BCD es un triangulo isosceles con BD =


DC. Si ABC = 160, determinar la medida de AEC.

Figura 13

4. (*) (XV Competencia de Clubes Cabri Primera Ronda) En la figura


adjunta, ABCD es un rectangulo tal que AB = 2BC. M es el punto
medio de AB y los triangulos AM E y M BF son equilateros. Si P
es la interseccion de las rectas DE y CF , encuentre los angulos del
4CDP .


5. Si AB y F G son rectas paralelas, el ABC = CDE = , el DEF = 2
y el GF H =
150. Calcule . Figura 14
6. Probar que una bisectriz exterior de un triangulo es paralela al lado opuesto si y solo si
el triangulo es isosceles.

11
Figura 14

7. (*) Hallar la suma de los angulos +  + + en la figura 15.

Figura 15

8. Determine el valor de la suma A + B + I + H + F + G. Figura 16.

Figura 16

12
9. En el 4ABC el BAC = 36 y AC = AB. Probar que la bisectriz interior BD, D en
AC, es congruente con el lado BC.

10. Sea ABC un triangulo rectangulo en B con AB = BC, se construye exteriormente el


triangulo equilatero BCD. Encuentre el angulo DAB.

11. En el 4ABC, AB = AC y D un punto sobre la recta AC, tal que BC = BD = DA.


Determine la medida del angulo ABD, si:

a) D esta entre A y C.
b) A esta entre D y C.

12. En un 4ABC, D es un punto sobre el lado AC tal que AB = AD. Si ABC ACB =
90, hallar el CBD.

13. En la figura 17, el ABC = ACE, DC = EC, Que lnea notable es AD del 4BCA?

Figura 17

14. Se tiene un triangulo isosceles ABC, AB = BC en el cual se traza al altura AF tal que
BF = 6 y F C = 2. Hallar AC.

15. Cual es el valor de b a en la figura 18?

Figura 18

13
16. La hipotenusa BC de un triangulo rectangulo ABC se divide en 4 segmentos congruentes
por los puntos G, E y H. Si BC = 20, encuentra la suma de los cuadrados de las longitudes
de los segmentos AG, AE y AH. Figura 19.

Figura 19

17. (*) Dado un cuadrado ABCD, se construyen los triangulos equilateros ABP (exterior-
mente) y ADQ (interiormente). Probar que C, P y Q estan alineados.

18. (*) Sea ABC un triangulo rectangulo con CAB = 90. D es un punto sobre la prolon-
gacion de BC tal que BD = BA. E es un punto en el mismo semiplano que A respecto
de BC, tal que CE BC y ademas CE = CA. Mostrar que A, D y E estan alineados.

19. El cuadrilatero ABCD mostrado en la figura 20 cumple que AB k CD y BC k DA.6


Sobre las prolongaciones de AB y AD se construyen puntos E y F tales que BC = BE
y DC = DF . Demuestre que C, E y F estan alinedos.

Figura 20

20. (*) En la figura adjunta, AB = BC = CD =


DE = EF = F G = GA. Calcule la medida del
DAE.

6
El cuadrilatero ABCD es un paralelogramo.

14
21. (*) (XXVIII Olimpiada Brasilena de Matematica) En la figura 21, AB = AC, AM = AN
y CAM = 30, encuentre el valor del BM N .

Figura 21

22. Los lados de un triangulo isosceles son 12 y 5 metros, cual es su permetro?


a+b+c
23. Muestre que los lados de un triangulo cumplen que |a b| < c y que c < 2
.

24. Muestre que es posible construir un triangulo con segmentos de longitudes a, b, c si y solo
existen numeros positivos x, y, z tales que: a = x + y, b = y + z, c = z + x.

Problemas de Refuerzo.

25. (*) (Etapa semifinal Estatal de XXII Olimpiada Mexicana de Matematicas) En la figura
22 se muestra un hexagono regular ABCDEF de lado 1. Los arcos del crculo que estan
dibujados tienen centro en cada vertice del hexagono y radio igual a la distancia al vertice
opuesto. P , Q, R, S, T y U son los puntos de corte de estos arcos. Cuanto mide cada
lado del hexagono P QRST U ?

Figura 22

15
3. Congruencia de Triangulos.
CRITERIOS DE CONGRUENCIA.
Definicion de Congruencia de triangulos. El 4ABC es congruente al 4A0 B 0 C 0 si: AB =
A0 B 0 , AC = A0 C 0 , BC = B 0 C 0 , ABC = A0 B 0 C 0 , ACB = A0 C 0 B 0 y BAC = B 0 A0 C 0 .
Simbolicamente: 4ABC = 4A0 B 0 C 0 . Vease figura 23.

Figura 23: Definicion de Igualdad de Triangulos.

La definicion anterior establece que dos triangulos son congruentes si tanto los lados como
los angulos se presentan en pares respectivos congruentes. Esto, segun la vision de Euclides,
significa que un triangulo es posible superponerlo sobre el otro (se puede desplazar, girar o
reflejar) y coincidira de manera perfecta. Sin embargo, es importante mencionar que en muy
raras ocasiones se tendra a disposicion tanta informacion, de all la importancia de los criterios
de congruencia, que establecen los requisitos mnimos para garantizar que dos triangulos son
congruentes.

El siguiente es el primero de los tres criterios de congruencia de triangulos, y se denomina


criterio de LADO-ANGULO-LADO, en smbolos: L-A-L.

Criterio L-A-L. Si los triangulos ABC y A0 B 0 C 0 presentan las congruencias: AB = A0 B 0 ,


AC = A0 C 0 y BAC = B 0 A0 C 0 , entonces 4ABC = 4A0 B 0 C 0 .

Figura 24: Criterio LAL

Segun el criterio L-A-L, dos triangulos son congruentes si en uno de ellos existen dos lados y el
angulo (comprendido entre dichos lados), respectivamente congruentes a dos lados y el angulo
(comprendido entre dichos lados), en el otro triangulo.

Criterio A-L-A. Sean ABC y A0 B 0 C 0 dos triangulos tales que: AC = A0 C 0 , BCA = B 0 C 0 A0


y BAC = B 0 A0 C 0 , entonces 4ABC = 4A0 B 0 C 0 .

16
Figura 25: Criterio ALA.

Criterio L-L-L. Si un triangulo tiene sus tres lados respectivamente congruentes a los tres
lados de otro triangulo, entonces estos dos triangulos son congruentes.

Figura 26: Criterio LLL.

Ahora demostraremos el Recproco del Teorema de Pitagoras.


Demostracion: Sea ABC un triangulo talque BC 2 = AB 2 + AC 2 , por construccion sea el
4A0 B 0 C 0 rectangulo en A0 tal que A0 B 0 = AB y A0 C 0 = AC, entonces por el teorema de
Pitagoras B 0 C 02 = A0 B 02 + A0 C 02 , as que B 0 C 02 = BC 2 , de donde B 0 C 0 = BC y por el criterio
LLL, se deduce que el 4A0 B 0 C 0 = 4ABC, por lo tanto el BAC = B 0 A0 C 0 = 90.

TEOREMA DE LA BASE MEDIA


En todo triangulo, el segmento que une los puntos medios de dos lados es paralelo al tercer
lado e igual a su mitad.

En la figura 27, M N es el segmento que une los puntos medios de los lados AB y BC del 4ABC,
AC
a este segmento se le llama BASE MEDIA DEL TRIANGULO. Se verifica que M N = y
2
que M N k AC.
Demostracion:
1. Prolongar el segmento M N hasta el punto P tal que M N = N P .
2. Los triangulos M N B y P N C son congruentes, ya que BN = N C, M N = N P y el
P N C = BN M , por consiguiente, el N CP = M BN , por lo tanto, CP k M B (Por
angulos alternos internos iguales). Ademas, P C = M B = M A; con lo cual se tiene que:
M A = P C.

17
Figura 27: Teorema de La Base Media.

Figura 28: Menor Media en un Triangulo Rectangulo.

3. Uniendo el punto A con el punto P se forman los triangulos congruentes AM P y ACP


(por L A L) ya que M A = P C, AP = AP , M AP = AP C (por angulos alternos
internos entre las paralelas M A y P C). Luego, M P = AC, entonces N P = 12 M P = 12 AC.
Ademas, P AC = M P A, de donde M P k AC o que M N k AC.

Corolario: Menor mediana de un triangulo rectangulo. En todo triangulo rectangulo,


la mediana relativa a la hipotenusa es la mitad de la longitud de la hipotenusa y es la menor
de las tres medianas del triangulo.

Demostracion: En la figura 28, BM es la mediana relativa a la hipotenusa AC del 4ABC,


probaremos que BM = AC 2
; (con lo cual se tendra que BM = AM = M C). Si por M se traza
una paralela al lado AB, que corte al lado BC en N , entonces N es el punto medio de BC y
el M N C = 90, los triangulos BN M y CN M son congruentes por el criterio L-A-L, luego
M B = M C = AM .
Probar que BM es la menor mediana (Ejercicio).

18
Ejercicios.

1. (*) En la figura adjunta, ABC es un triangulo equilatero


y CDEF es un cuadrado. Se construye un punto G tal
que CF = CG y ademas CF G = 15. Probar que
AGC = BDC.

2. Dado un triangulo equilatero ABC, se construye un triangulo equilatero DEF cuyos


vertices estan sobre los lados del 4ABC, tal como muestra la figura 29. Demuestre que
los triangulos ADF , BED, CF E son todos congruentes entre si.

Figura 29

3. ABCD es un cuadrado, E, F , G y H son puntos sobre los lados AB, BC, CD, DA, res-
pectivamente, tal que EF GH tambien es cuadrado. Demuestre que los triangulos AEH,
BF E, CGF , DHG son todos congruentes entre si. Figura 30.

Figura 30

19
4. ABCDE y F GHIJ son pentagonos regulares (Vease figura 31). Demuestre que los
triangulos AF J, BGF , CHG, DIH, EJI son todos congruentes entre si.

Figura 31

5. Si AB k CD y AB = CD entonces, AD = BC y AD k BC 7 .

6. Demuestre que dos triangulos desplazados


son congruentes. Sugencia: Utilice el pro-
blema anterior.

7. Demuestre que dos triangulos rotados son congruen-


tes.

7
El cuadrilatero ABCD se denomina paralelogramo.

20
8
8. Demuestre que dos triangulos reflejados con respecto a un punto son congruentes.

9. Demuestre que dos triangulos reflejados con respecto a


una recta son congruentes.

Importante: Las traslaciones, rotaciones y reflexiones no cambian el tamano ni la forma


de un triangulo.

10. (*) En la figura adjunta, ABCD un cuadrado y EF GH.


Demuestre que que EF = GH.

11. Dos cuadrados ABCD y EHGF , ambos de lado l,


estan colocados en manera tal que un vertice de uno
esta en el centro del otro (como en la figura anexa).
l2
Demuestre que el area del cuadrilatero EJBK es
4
y por ende no depende de la posicion de J (o K).

8
La reflexion con respecto a un punto es equivalente a una rotacion de 180

21
12. En un 4ABC el B = 2C, la mediatriz del lado AC corta en F al lado BC. Hallar
AB, si F C = 9.

13. (*) (Examen final de XVI Olimpiada mexicana de Matematica) Los angulos de un triangu-
lo ABC estan en progresion aritmetica (B A = C B = ), D, E, y F son los
puntos medios de los lados BC, CA y AB, respectivamente. Llamamos H al pie de la
altura trazada desde C (que cae entre B y F ) y G a la interseccion entre DH y EF .
Cuanto vale F GH?

14. En la figura 32, AC = 12 AF = 4 y BAF = 30. Hallar BF si AG = GC.

Figura 32

15. En la figura 33, AG = GC, el AF G = 20. Hallar el F AC, si AC = 2BF .

Figura 33

16. (*) Sea ABCD un cuadrado. Se construyen triangulos equilateros ADP y ABQ como se
muestra en la figura 34. Sea M la interseccion de CQ con AD y N la interseccion de CP
con AB. Demuestre que CM N es un triangulo equilatero.

22
Figura 34

Problemas de Refuerzo.

17. En la figura 35, ABC, CDE y EF A son triangulos isosceles, con el ABC = CDE =
EF A = 120. Probar que el 4BDF es equilatero.

Figura 35

18. (*) 4ABC es un triangulo isosceles con ABC = ACB = 80. D es un punto en AC
tal que ABD = 10. Demuestre que AD = BC.

23
4. Cuadrilateros: Clasificacion y Propiedades.
CLASIFICACION.
Los cuadrilateros pueden clasificarse de acuerdo a sus diagonales de la siguiente forma:

Cuadrilatero Convexo: Es un cuadrilatero con las dos diagonales en su interior.

Cuadrilatero Entrante: Es un cuadrilatero con una diagonal en el interior y otra en el exte-


rior.

Cuadrilatero Cruzado Es un cuadrilatero con las diagonales en su exterior.9

Es muy frecuente que se considere que un cuadrilatero es convexo, a menos que se especifique lo
contrario. Esto es as porque muchos resultados son mas claros en un cuadrilatero convexo, sin
embargo, es importante darse cuenta que existen teoremas que no se cumplen para cualquier
tipo de cuadrilateros, por ejemplo:

Teorema: La suma de los angulos internos de un cuadrilatero no cruzado es 360.

La demostracion de este resultado se basa en la diseccion del cuadrilatero en dos triangu-


los cuyos angulos internos conforman los angulos internos del cuadrilatero, sin embargo, estas
condiciones no pueden lograrse en un cuadrilatero cruzado; de hecho, la suma de los angulos
internos puede hacerse arbitrariamente pequena cuando el cuadrilatero es cruzado.

Tambien hay otras clasificaciones de cuadrilateros de acuerdo a sus lados y angulos.

Cuadrilatero Equiangulo: un cuadrilatero (convexo) es equiangulo si todos sus angulos inter-


nos son iguales; dado el teorema anterior, los angulos son iguales a 90, por ello este cuadrilatero
es llamado rectangulo.

Cuadrilatero Equilatero: un cuadrilatero (convexo) es equilatero si todos sus lados son igua-
les. A este cuadiratero tambien se le conoce como rombo.

Cuadrado: es un cuadrilatero que es equiangulo y equilatero.

Paralelogramo: es un cuadrilatero con los lados opuestos paralelos.

Trapecio: es un cuadrilatero con un par de lados opuestos paralelos.10

9
Tanto los cuadrilateros convexos como los entrantes son cuadrilateros simples, que son los cuadrilateros
cuyos lados no se cortan salvo en los extrenos; en contraposicion, los cuadrilateros cruzados no son simples.
10
Note que un paralelogramo es tambien un trapecio.

24
PARALELOGRAMOS
Dado el paralogramo ABCD, por propiedades de angulos entre paralelas es posible probar el
siguiente resultado:

Teorema: Los angulos opuestos son iguales y los angulos consecutivos son suplementarios:
ABC = CDA = y BCD = DAB = 180 .

Por otra parte, por criterio ALA, 4ABC 4CDA; esto implica que AB = CD y BC = DA,
i.e.

Teorema: Los lados opuestos de un paralogramos son iguales.

A partir de esto, si M es la interseccion de AC con BD, por criterio ALA, 4ABM 4CDM ,
por lo que AM = CM y BM = DM , i.e.

Teorema: Las diagonales de un paralelogramo se bisecan.

Ademas, se cumple un resultado sofisticado y muy importante:

Teorema: Ley del Paralelogramo. Si ABCD es un paralelogramo entonces el doble de la


suma de los cuadrados de los lados es igual a la suma de los cuadrados de las diagonales, es
decir
2 AB 2 + BC 2 = AC 2 + BD2


Demostracion: Aplicando la Ley del Coseno a 4ABC y 4ABD se tiene

AC 2 = AB 2 + BC 2 AB BC cos
DB 2 = AB 2 + AD2 AB AD cos(180 )
AC 2 + DB 2 = 2 AB 2 + BC 2 AB BC (cos + cos(180 ))


y dado que cos = cos(180 ) el resultado se sigue inmediatamente.

RECTANGULOS
En primer lugar, es importante notar que todo rectangulo es paralelogramo (por angulos en-
tre paralelas), por lo que todos los resultados probados anteriormente son heredados a todo
rectangulo; pero los rectangulos tienen propiedades adicionales:

Observe que por criterio LAL, 4ABC 4ABD, por lo que AC = BD y entonces
Teorema: Las diagonales de un paralelogramo son iguales; ademas, el punto de interseccion
de estas equidista de los cuatro vertices y por tanto es el centro de una circunferencia que pasa
por todos los vertices.

Por otra parte, observe que si se aplica la ley del paralelogramo a un rectangulo se obtiene el
Teorema de Pitagoras.

25
ROMBOS
Dado un rombo ABCD, por criterio LLL, 4ABC 4CDA, y por lo tanto BAC = DAC
y BCA = DAC, lo cual implica BC k AD y AB k CD, i.e., todo rombo ABCD es un
paralelogramo. Ademas, por las mismas congruencias se tiene

Teorema: Las diagonales de un rombo cumplen ser una mediatriz de la otra.

Teorema: Las diagonales de un rombo bisecan a los angulos interiores del rombo; esto implica
que el punto de corte de las diagonales equidista de los cuatro lados del rombo y es el centro
de una circunferencia tangente a estos.

TRAPECIOS
Dado el trapecio ABCD (con AB k CD), se construyen los puntos medios de BC y DA, M y
N , respectivamente. Si el cuadrilatero M N AB se rota con centro en M y angulo 180 se genera
un cuadrilatero M N 0 A0 C; observe que N D = N 0 A0 y N D k N 0 A0 , por lo que DN N 0 A0 es un
paralelogramo y

N N 0 = DA0
2M N = DC + CA0
2M N = DC + AB
AB + CD
MN =
2
El segmento M N es llamado base media del trapecio, y por lo recien demostrado se tiene

Teorema: La base media de un trapecio es igual a la semisuma de las bases.

Por otra parte, hay ciertos trapecios que reciben nombres particulares; el trapecio rectangulo es
aquel que las bases son perpendiculares a alguno de los otros lados; y por otra parte, el trapecio
isosceles es aquel que los lados (distintos de las bases) tienen igual longitud. 11

Ejercicios
1. Dado el trapecio ABCD con AB k CD, demuestre que la bisectriz interior del A es
paralela a la bisectriz exterior del D.

2. A un rombo ABCD se le construyen exteriormente los cuadrados ABEF y BCGH.


Demuestre que 4ABD = 4EBH.

3. (*) Sea ABCD un paralelogramo. Se construyen triangulos equilateros exteriores 4CDP


y 4ADQ. Demuestre que el 4BP Q es equilatero.

4. Demuestre que las bisectrices interiores de un paralelogramo forman un rectangulo (que su-
cede si el paralelogramo es ademas rombo?).
11
Los trapecios isosceles son muy importantes cuando se estudian los angulos en la circunferencia; resulta que
un trapecio es isosceles si y solo si los cuatro vertices se ubican sobre una misma circunferencia.

26
5. Demuestre que las bisectrices exteriores de un paralelogramo forman un rectangulo.

6. Sea ABCD un paralelogramo. La bisectriz interna del CDA corta a BA en M , y la


bisectriz interna del BAD corta a CD en N . Demuestre que ADN M es un rombo.

7. Demuestre que si por el punto de interseccion de las diagonales de un rombo se tra-


zan perpendiculares a los lados del rombo, entonces los puntos de interseccion de dichas
perpendiculares con los lados del rombo forman un rectangulo.

8. Demuestre que las bisectrices de los angulos definidos por las diagonales de un rombo,
cortan a los lados del rombo en cuatro puntos que forman un cuadrado.

9. En un 4ABC sea G la interseccion de las medianas BB 0 y CC 0 . Sean B 00 , C 00 las reflexiones


de G respectivas a los puntos B 0 y C 0 .

a) Demuestre que AGCB 00 y AGBC 00 son paralelogramos.


b) A partir de lo anterior, demuestre que BCB 00 C 00 tambien es paralelogramo.
c) Demuestre que A0 pertenece a la recta AG, y concluya que las tres medianas de un
triangulo concurren en el punto G, llamado el centroide del 4ABC.
d) Demuestre que CG = 2GC 0 ; relaciones similares se cumplen para las otras dos media-
nas.

10. Teorema de Varignon: Dado un cuadrilatero ABCD (no necesariamente convexo), se


construyen los puntos medios L, M , N , O, P , Q, de los segmentos de recta AB, BC, CD,
DA, BD, AC, respectivamente. Figura 36.

a) Demuestre que LM N O, LP N Q, OP M Q, son paralelogramos.


b) Demuestre que LN , OM , P Q concurren en un punto, llamado el centroide del cua-
drilatero ABCD.
c) Demuestre que el permetro de LM N O es igual a AC + BD; resultados similares se
cumplen para los otros paralelogramos.

Figura 36: Teorema de Varignon

27
11. Sea ABCD un paralelogramo tal que existe un punto E sobre el lado AB que cumple
CED = 90. Sean M y N los pies de las perpendiculares trazadas desde A y B hacia
DE y CE, respectivamente. Demuestre que AC, BD y M N concurren.

12. (*) (Hector Alberti) Sea ABCD un cuadrado. Se construyen los triangulos equilateros
BDA0 , ACB 0 , BDC 0 y ACD0 . Demuestre que el A0 B 0 C 0 D0 es tambien un cuadrado.

13. (*) (II Olimpiada Matematica del Cono Sur) En la figura 37 ABCD y AECF son para-
lelogramos. Demuestre que BEDF es paralelogramo.

Figura 37

Problemas de Refuerzo.

14. (*) ABCD es un cuadrilatero convexo y O es un punto en su interior. Sean P , Q, R, S,


los puntos medios de los lados AB, BC, CD, DA, respectivamente. Por P se traza una
paralela a OR, por Q se traza una paralela a OS, por R se traza una paralela a OP , y
por S se traza una paralela a OQ. Demuestre que estas cuatro rectas concurren.

15. (*) Un trapecio isosceles tiene diagonales perpendiculares y su area es 2010, determine su
altura.

16. (*) (IX Competencia de Clubes Cabri, Segunda Ronda) Sea ABCDEF un hexagono
regular cuyo centro es O. Se construyen los cuadrados F SOP y ORCQ. Demuestre que
AP QB y SEDR son rectangulos. Figura 38.

17. (*) Sobre los lados del 4ABC se trazan exteriormente los cuadrados ABP Q, CARS y
BCT U . Luego se trazan los paralelogramos AQA0 R, CSC 0 T y BU B 0 P .

a) Sean A00 , B 00 , C 00 los centros de los cuadrados BCT U , CARS, ABP Q, respectiva-
mente. Demuestre que estos centros estan sobre los lados del 4A0 B 0 C 0 .
b) Demuestre que AA00 , BB 00 , CC 00 concurren.

28
Figura 38

18. (*) Se dibujan cuadrados exteriores a los lados de un paralelogramo, demuestre que:

a) El cuadrilatero determinado por los centros de esos cuadrados es un cuadrado.


b) Las diagonales de ese cuadrado son concurrentes con las del paralelogramo.

19. (*) Dado un 4ABC, se construyen exteriormente los triangulos rectangulo isosceles
4ACP y 4BCQ, con AC y BC como hipotenusas. Si M es el punto medio de AB,
demuestre que el 4M P Q tambien es un triangulo rectangulo isosceles.

29
5. Angulos en la Circunferencia.
LA CIRCUNFERENCIA Y SUS ELEMENTOS
Una circunferencia es el lugar geometrico de puntos que equidistan de un punto dado, llamado
el centro de la circunferencia; la distancia de cada punto de la circunferencia al centro es el radio.

Por otra parte, todos los puntos que estan a una distancia del centro menor o igual al radio
forman el crculo; estos puntos quedan al interior o sobre la circunferencia.

Si A y B son dos puntos de una circunferencia, el segmento de recta AB define una cuerda; en
particular, si el centro de la circunferencia pertenece a la cuerda, esta es llamada diametro. Es
importante mencionar que para cada punto de la circunferencia existe exactamente un punto
diametralmente opuesto.

En la figura 39, se tiene una circunferencia de centro O y radio r = OA = OB = OA0 ; AB


y AA0 son cuerdas, pero AA0 es tambien diametro, i.e, A0 es diametralmente opuesto a A y
viceversa. Observe que por la desigualdad triangular aplicada al triangulo isosceles 4AOB

Figura 39

AB < AO + BO
= r+r
= AA0
Si A es un punto fijo, esta desigualdad es valida para cualquier punto B sobre la circunferencia
(excepto cuando B = A0 lo cual implica AB = AA0 ). Esto quiere decir que el diametro es la
mayor de todas las cuerdas.
A las porciones de circunferencia que quedan entre dos puntos ubicados en la circunferencia,
se les llama arcos de circunferencia; note que dos puntos sobre una circunferencia definen dos
arcos de circunferencia. Tambien, si un angulo tiene vertice sobre el centro de la circunferencia y
esta formado por dos radios, sera llamado angulo central ; de nuevo, AOB hace referencia a dos
angulos, cuya suma es 360, y subtienden respectivamente a uno de los arcos AB. Finalmente,
si un angulo tiene el vertice sobre la circunferencia y esta formado por dos cuerdas, sera llama-
do angulo inscrito; en la figura anterior, AA0 B es un angulo inscrito que subtiende al arco AB.

30
Teorema: El angulo central es el doble del angulo inscrito que subtiende el mismo arco.

Demostracion: Considere la figura 40, se demostrara que AOB = 2AP B en los tres casos
mostrados. En la circunferencia de la izquierda, sea P 0 el punto diametralmente opuesto a P ;
observe que 4AP O y 4BP O son triangulos isosceles, y por el teorema del angulo externo se
tiene

AOB = AOP 0 + BOP 0


= (AP O + OAP ) + (BP O + OBP )
= 2AP O + 2BP O
= 2 (AP O + BP O)
= 2AP B

Figura 40

El caso de la circunferencia del medio es mas sencillo y se deja como ejercicio para el lector.
Para la circunferencia de la derecha, el trabajo es analogo y solo cambia en un pequeno arreglo
algebraico

AOB = BOP 0 AOP 0


= (BP O + OBP ) (AP O + OAP )
= 2BP O 2AP O
= 2 (BP O AP O)
= 2AP B

Corolario: Todos los angulos inscritos que subtienden el mismo arco son iguales (Ver figura
41). En particular, los angulos internos son iguales a 90 si subtienden a una semicircunferencia.

Demostracion: Todos los angulos mostrados en la figura 41 son iguales a la mitad del AOB,
y por tanto, son iguales entre s. En particular, si AB fuera un diametro, AOB = 180 y por
tanto AP B = 90. 12

Hay un par de angulos mas que son importantes: Si un punto P es interno a la circunferencia,
el angulo de vertice P formado por dos cuerdas que pasan por P se llama angulo interior. De
12
Observe que en cualquier triangulo rectangulo, el punto medio de la hipotenusa equidista de los tres vertices.

31
Figura 41

forma similar, si P es exterior y dos cuerdas de la circunferencia (al prolongarse) pasan por P ,
el angulo con vertice P es llamado angulo exterior.

Dejamos como ejercicio demostrar el siguiente teorema:

Teorema: Los angulos interior y exterior mostrados en la figura 42 cumplen las formulas
siguientes:
BOD + AOC
AQC =
2
BOD AOC
AP C =
2

Figura 42

CUADRILATEROS CICLICOS
Ahora suponga que sobre una circunferencia se ubican cuatro puntos A, B, C, D, como se
muestra en la figura 43. Al cuadrilatero ABCD se le llama cuadrilatero cclico o concclico.
Observe que

ABC + CDA = + = 180 .
2 2

32
Figura 43

Y analogamente DAB +BCD = 180. Esto significa que si ABCD es un cuadrilatero cclico
y convexo, entonces los angulos opuestos son suplementarios. Tambien, es posible demostrar por
contradiccion el recproco de este resultado: si suponemos que ABCD es tal que B+D = 180
pero no es cclico, se define el punto D0 como la otra interseccion de AD con el circuncrculo
del 4ABC, y como ABCD0 es cclico (por construccion) entonces B + D0 = 180, luego,
D = D0 , lo cual implica la contradiccion CD k CD0 (rectas paralelas que se cortan en C).
As, se ha demostrado el siguiente teorema:

Teorema: El cuadrilatero convexo ABCD es un cuadrilatero cclico si y solo si

A + C = 180 = B + D

Tambien, otro criterio muy util y cuya demostracion tambien se basa en el corolario anterior es

Teorema: El cuadrilatero convexo ABCD es un cuadrilatero cclico si y solo si se cumple


alguna de las siguientes igualdades

ABD = ACD
BCA = BDA
BAC = BDC
CAD = CBD

Es importante recalcar que NO todo cuadrilatero puede ser inscrito en una circunferencia; por
ejemplo, un paralelogramo no sera cclico a menos que sea rectangulo.

RECTAS Y CIRCUNFERENCIAS TANGENTES A UNA CIRCUN-


FERENCIA
Dada una circunferencia, una recta puede ser tangente o secante a la circunferencia, depen-
diendo si la corta en uno o dos puntos, respectivamente; en cualquier otro caso, se dice que la

33
recta no corta a la circunferencia.13

Sea l una recta secante a la circunferencia que corta a la circunferencia en A y B (A 6= B);


como el 4AOB es isosceles, OAB < 90. Recprocamente, si por A se traza una recta l tal
que uno de los angulos que forma con OA es menor que 90, se puede construir un punto B
sobre l tal que OAB = ABO < 90 y A 6= B (basta proyectar O sobre l y luego reflejar A
con respecto a este punto, el resultante es el punto B); entonces el 4AOB es isosceles, por lo
que OA = r = OB, i.e. B pertenece a la circunferencia y por tanto l corta a la circunferencia
en dos puntos distintos. As

Teorema: Una recta l corta a una circunferencia de centro O en dos puntos distintos A y B si
y solo si un angulo entre l y OA es agudo.

Corolario: Si l es una recta tangente en A a una circunferencia de centro O, ninguno de los


angulos entre l y OA puede ser agudo, y por tanto l OA.

A partir de este resultado se prueban otros resultados muy conocidos y utiles, que dejamos de
ejercicios para el lector.

Teorema: Dado un punto P externo a una circunferencia de centro O, si P A y P B son segmen-


tos tangentes a la circunferencia en A y B, respectivamente, entonces el cuadrilatero P AOB es
cclico y bisosceles.

Corolario: Dado un punto P externo a una circunferencia de centro O, la circunferencia de


diametro P O corta a la circunferencia dada en dos puntos A y B tales que P A y P B son rectas
tangentes.

Definicion: El angulo semi-inscrito en una circunferencia es aquel que se forma con una cuerda
y la recta tangente en alguno de los extremos de la cuerda.

Teorema: La media del angulo semi-inscrito definido por la cuerda AB es igual a la medida
de un angulo inscrito que subtiende al arco AB.

Demostracion: Considere la figura 44. Como AP BO es cclico, entonces P AB = P OB;


ademas, como P O es la mediatiz de AB, P OB = P OA, por lo que
AOB
P AB = = AQB
2
Por otra parte, dada una circunferencia, otra circunferencia puede ser secante o tangente a la
primera, dependiendo si la corta en uno o dos puntos, respectivamente; en cualquier otro caso
se dice que las circunferencias no se cortan.14

13
Cuando la recta es tangente a la circunferencia puede considerarse como un caso muy peculiar en el cual
los dos puntos de corte coinciden.
14
Tambien aca puede considerarse a las circunferencias tangentes como un caso especial de circunferencias
secantes en el cual los puntos de corte coinciden.

34
Figura 44

Ademas, dos circunferencias pueden posicionarse una dentro de la otra, y claramente, la cir-
cunferencia de radio mayor es la externa mientras que otra es la interna; particularmente, si las
circunferencias tienen el mismo centro se llaman concentricas. Finalmente, combinando estas
definciones se tienen las circunferencias tangentes exteriormente y las tangentes interiormente.

Teorema: Dadas dos circunferencias de centros O1 y O2 que se cortan en dos puntos distintos
A y B, se cumple que O1 O2 AB.

Teorema: Si dos circunferencias de centros O1 y O2 son tangentes en A, se cumple que O1 , A


y O2 estan alineados.

Teorema:

a) Dos circunferencias, una dentro de la otra, no tienen rectas tangentes en comun.

b) Dos circunferencias tangentes interiormente tienen una recta tangente comun.

c) Dos circunferencias secantes (en dos puntos distintos) tienen dos rectas tangentes en comun.

d) Dos circunferencias tangentes exteriormente tienen tres rectas tangentes en comun.

e) Dos circunferencias no secantes y tal que ninguna contiene a la otra, tienen cuatro rectas
tangentes en comun.

Ejercicios

1. Si el M P Q = 20, determine el valor del QON en la figura


adjunta.

35
2. Dado un angulo inscrito BAC, y su angulo central BOC, se sabe que BAC + BOC =
180. Calcular el OBC.

3. En la figura 45, BCDO es un rombo. Determine el valor del angulo y la medida de las
diagonales de BCDO si el radio de la circunferencia mide 6.

Figura 45

4. Un cuadrilatero cclico ABCD satisface ABC = 2CDA = . Calcule .

5. En la figura 46, P R es una tangente comun. Calcule el valor del P QR.

Figura 46

6. En la figura adjunta, el AF E = 100 y el BCD =


150. Calcule el AGB.

7. Dado un angulo AOB, se trazan dos rectas l OA y m OB. Si P es el punto de


corte de l y m, demuestre que A, B, O, P se ubican sobre una misma circunferencia.

36
8. Las bisectrices BP y CQ del 4ABC se cortan en I. Demuestre que si BAC = 60
entonces 4P QI es isosceles.
9. En la figura 47 se ha tomado un punto C sobre la circunferencia; AC y BC cortan a la
segunda circunferencia en D y E respectivamente. Probar que OC DE.

Figura 47

10. (*) Dada la figura 48, demuestre que AB k A0 B 0 .

Figura 48

11. En la figura 49 CR es una recta tangente en C, demuestre que AB k CR.

Figura 49

37
12. Dos circunferencias 1 y 2 son tangentes (interior o exteriormente) en P (Ver figura 50).
Dos rectas que pasan por P cortan a 1 y 2 en A y C, y en B y D, respectivamente.
Demuestre que AB k CD.

Figura 50

13. (*) Dos circunferencias de centros O1 y O2 son tangentes (interna o externamente) en un


punto P ; por este punto se traza una recta que corta nuevamente a la circunferencias en
A y B, respectivamente. Demuestre que AO1 k BO2 .

14. Dos circunferencias son tangentes externamente en el punto A. Una tangente exterior
comun toca a una circunferencia en B y a la otra en C. Demostrar que BAC = 90.

15. En la figura 51, DE es tangente en D, y C es el punto medio del arco AD. Encuentre el
valor del angulo seminscrito ADE.

Figura 51

38
16. Determine el valor del DCF , sabiendo BE es tangente en el punto D a la circunferencia
de centro O. Ver Figura 52.

Figura 52

17. Si el AEB = 30, ADE = 20 y ACE = 35, calcule el AF B. Vease figura 53.

Figura 53

18. Dada una circunferencia de diametro BC, se toma un punto P en la prolongacion de BC,
y se traza la tangente AP . Si AP = AB y O es el centro de la circunferencia, demuestre
que el 4AOC es equilatero.
19. (*) Dadas dos circunferencias una fuera de la otra, demuestre que las tangentes comunes
externas forman segmentos iguales; analogamente, las tangentes comunes internas forman
segmentos iguales.
20. (*) Teorema de Pithot. Demuestre que en todo cuadrilatero inscribible, la suma de
lados opuestos es igual.
21. (*) Teorema de Steiner. En todo cuadrilatero exinscrito a una circunferencia, la dife-
rencia de las longitudes de lados opuestos es igual.

39
22. Demuestre que las mediatrices de un cuadrilatero son concurrentes si y solo si es cclico.
23. Demuestre que el cuadrilatero convexo ABCD es inscribible si y solo si los incrculos
respectivos del 4ABC y 4CDA son tangentes.
24. Demuestre que las bisectrices internas de un cuadrilatero son concurrentes si y solo si es
inscribible.
25. Demuestre que todo rombo es inscribible.
26. En la figura 54, AB es una cuerda y por D se traza una recta tangente a la circunferencia
paralela a AB. Demuestre que CD es bisectriz del ACB.

Figura 54

27. Determine las medidas de ACB y ACO de la figura 55.

Figura 55

28. Cuatro cilindros de diametro 1 estan pegados apretadamente por


una cuerda muy fina, como en la figura adjunta. Demostrar que la
cuerda tine longitud 4 + . Demostrar tambien que el area som-
breada entre los cilindros es 1 4 .

29. En la figura 56, ABCD es un trapecio isosceles con AB k CD y DA = BC = 2; tomando


DA y BC como diametros, se construyen dos circunferencias tangentes. Si DC = 3AB,
calcule el area del trapecio.

40
Figura 56

30. La figura 57 esta formada por un paralelogramo y dos circunferencia tangentes entre s y
tangentes a tres lados del paralelogramo. Sabiendo que el radio de las mismas mide la
cuarta parte del lado menor del paralelogramo, calcule la razon entre el lado mayor del
paralelogramo y el radio de las circunferencias.

Figura 57

31. En la figura 58, ABCDEF es un hexagono regular y las circunferencias de centro en los
vertices son tangentes dos a dos. Si las circunferencias sobre los vertices B, D, F son
iguales, demuestre que las circunferencias restantes son iguales.

Figura 58

32. Alrededor de una circunferencia se construyen diez circunferencias tangentes a la original


y tangentes entre s (Vease figura 59). Demuestre que la suma de las areas de las diez
circunferencias es el doble del area de la circunferencia mayor.

41
Figura 59

33. (*) Teorema de Miquel: Dado un 4ABC, sean X, Y , Z puntos sobre AB, BC, CA,
respectivamente. Demuestre que los circuncrculos de 4AXZ, 4BY X, 4CZY tienen un
punto en comun M .

34. (X OMCC - P2, Aaron) Sea ABCD un cuadrilatero concclico con diametro AC, y sea
O el centro de su circunferencia. Se construyen los paralelogramos DAOE y BCOF .
Demuestre que si E y F estan sobre la circunferencia entonces ABCD es rectangulo.

35. (*) Sea ABC un triangulo, y sean L y N las intersecciones de la bisectriz del angulo A
con el lado BC y el circuncrculo de ABC respectivamente. Construimos la interseccion
M del circuncrculo de ABL con el segmento AC. Prueba que los triangulos BM N y
BM C tienen la misma area.

36. (*) Sea AB el diametro de una semicircunferencia. Se colocan los puntos M y K sobre
la semicircunferencia y sobre AB, respectivamente.15 Sea P el centro de la circunferencia
que pasa por A, K y M ; sea Q el centro de la circunferencia que pasa por B, K y M .
Demuestre que M P KQ es concclico.

37. (*) Las circunferencias 1 y 2 se cortan en los puntos A y B. Por el punto A se traza
una recta que corta nuevamente a las circunferencias 1 y 2 en los puntos C y D,
respectivamente. Por los puntos C y D se trazan tangentes a las circunferencias, las
cuales se cortan en el punto M . Demuestra que M CBD es cclico.

38. (*) El 4ABC cumple que A = 90 y AB = AC. Se toma un punto E del segmento
AB, se construye interiormente un triangulo equilatero AEF . EF corta BC en I, y se
construye exteriormente un triangulo equilatero BIJ. Encuentre EJB.

39. (*) En la figura 60, se sabe que AO1 B AO2 B = 70 y ademas la tangente EB forma
el triangulo isosceles ABE, con AB = AE. Encuentre EBC.
15
M y K son distintos de A y B.

42
Figura 60

40. (*) Dos circunferencias 1 y 2 se cortan en A y B. Una recta por A corta a 1 y 2 en C


y D, respectivamente, y la paralela a CD por B corta 1 y 2 en E y F , respectivamente.
Demuestre que 4CDB 4EAF .
41. (*) La Recta de Simson-Wallace. Sean X, Y y Z los pies de las alturas trazadas
desde un punto P en el circuncrculo del 4ABC hacia AB, BC y CA, respectivamente.
Demuestre que X, Y y Z estan alineados.
42. (*) Sea P un punto exterior al cuadrado ABCD tal que AP C = 90 , Q es la interseccion
de AB y P C, y R el pie de la perpendicular por Q a CA. Demuestre que P , R y D estan
alineados.
43. En la figura 61, ABCD es un trapecio rectangulo tal que la circunferencia de diametro
AB (y centro O) es tangente a CD. Demostrar que O pertenece a la circunferencia de
diametro CD y que esta circunferencia es tangente a BA.

Figura 61

44. El 4ABC es rectangulo en C, la circunferencia de centro O es tangente a cada uno de los


lados del 4ABC en los puntos P , Q y R (como se muestra en la figura 62), y se cumple
que AP = 20 y BP = 6. Calcule OP .

43
Figura 62

45. En la figura 63 se muestran tres semicircunferencias, una de diametro AB (de centro O


y radio r), otra de diametro AO y la ultima de diametro OB. Determine la razon entre
el radio de la circunferencia tangente a estas tres semicircunferencias y r.

Figura 63

46. El segmento AB es diametro de un semicrculo con centro en O. Un crculo con centro


en P es tangente a AB en O y tambien al semicrculo. Otro crculo con centro en Q es
tangente a AB, al semicrculo y al crculo de centro en P . Si AB = 2, cual es el radio
del crculo con centro en Q?

Figura 64

Problemas de Refuerzo.

47. Los vertices A y B de un triangulo equilatero 4ABC estan sobre una circunferencia de
radio 1 y el vertice C esta en el interior de la circunferencia. Un punto D (distinto de B)
que esta en la circunferencia es tal que AD = AB. La recta DC corta por segunda vez a
la circunferencia en E. Encuentre la longitud del segmento CE. Ver figura 65.

48. (*) (OIM 2002, P-4) En un triangulo escaleno ABC se traza la bisectriz interior BD, con
D sobre AC. Sean E y F puntos sobre la recta BD tales que (AE k CF ) BD, y sea
M el punto sobre el lado BC tal que DM BC. Demuestre que EM D = DM F .

44
Figura 65

49. (*) (OMCC 2003, P-2) Sea S una circunferencia y AB un diametro de ella. Sea t la recta
tangente a S en B y considere dos puntos C y D en t tales que B este entre C y D. Sean
E y F las intersecciones de S con AC y AD y sean G y H las intersecciones de S con
CF y DE. Demuestre que AH = AG.

50. (*) (The 59th Romanian Mathematical Olympiad District Round) Considere un cuadrado
ABCD y un punto E sobre el lado AB. La diagonal AC corta al segmento DE en el punto
P . La perpendicular por P a DE corta al lado BC en F . Probar que EF = AE + CF .

51. (*) Teorema de Arqumedes: En la figura 66, la region delimitada por tres semi-
circunferencias mutuamente tangentes, es conocida como cuchilla de zapatero o arbelos.
Demostrar que las circunferencias sombreadas son congruentes.

Figura 66: Teorema de Arqumedes.

45
6. Teorema de Thales y su recproco. Semejanza de Triangu-
los.
Introduccion.
Definicion
1. Razon: se llama razon, al cociente de dos cantidades, expresadas en la misma magnitud,
por ejemplo ab .

2. Proporcion: se llama proporcion a la igualdad de dos razones. Por ejemplo ab = dc , 16 a los


terminos a y d se les llama extremos y los terminos b y c se les llama medios, al termino
d se le llama cuarta proporcional entre a, b y c en este orden.
Propiedades de las proporciones:
a c
1. = si y solo si a c = b d.
b d
a c b d a b
2. = si y solo si = o = .
b d a c c d
a c ab cd
3. = si y solo si = .
b d b d
a c a+b c+d
4. = si y solo si = .
b d ab cd

Paralelismo y proporcionalidad.
Definicion
PA
1. Un punto P AB divide al segmento AB en una razon dada r, si PB
= r.

Figura 67

2. Sean AB y CD dos segmentos, y sean P AB y Q CD, decimos que P y Q dividen a


QA
AB y CD en segementos proporcionales si PP B
A
= QB .

Figura 68

16
En algunos textos de geometra se utiliza la notacion de proporcion as a : b :: c : d que se lee a es a b como
c es a d.

46
Teorema de Thales. Si tres paralelas cortan a dos secantes entonces los segmentos que de-
terminan en ellas son proporcionales. 17

Antes de demostrar el Teorema de Thales, se enunciaran dos teoremas que a pesar de su apa-
rente sencillez es de mucha utilidad en problemas que involucran Areas y Proporcionalidad.

Lema 1. Sea AB k CD. Demuestre que: (ABC) = (ABD).

Lema 2. Sea P un punto sobre el lado AB (o su prolongacion) del 4ABC. Pruebe que:

AP (AP C)
=
PB (P BC)
.
A continuacion se enuncian los pasos a seguir en la demostracion del teorema de Thales.
Demostracion. Sean AA0 , BB 0 y CC 0 rectas paralelas que cortan a dos secantes en los puntos
A, A0 , B, B 0 , C, C 0 respectivamente (ver figura 69).

Figura 69: Teorema de Thales

Pruebe que:
AB (ABB 0 )
1. =
BC (BCB 0 )
A0 B 0 (A0 B 0 B)
2. 0 0 = .
BC (B 0 C 0 B)
3. (ABB 0 ) = (A0 B 0 B) y (BCB 0 ) = (B 0 C 0 B).
Con ayuda de las igualdades demostradas concluya que:
AB A0 B 0
= 0 0.
BC BC
Observacion Importante: Utilice las propiedades de las proporciones para demostrar las
equivalencias siguientes (interpretelas geometricamente):

AB A0 B 0 AC A0 C 0 AC A0 C 0
= 0 0 = 0 0 = 0 0
BC BC AB AB BC BC
17
El teorema de Thales puede enunciarse de manera general como sigue: Si tres o mas paralelas cortan a dos
o mas secantes entonces los segmentos que determinan en ellas son proporcionales.

47
Corolario (Teorema de Thales en el triangulo). Toda recta paralela a un lado de un triangulo
y que corte a los otros dos lados, divide a estos lados en segmentos proporcionales.

Recproco del Teorema de Thales. Si tres rectas cortan a dos secantes en segmentos pro-
porcionales y dos de estas rectas son paralelas entonces las tres rectas son paralelas.

Demostracion. Sean AA0 , BB 0 y CC 0 rectas que cortan a dos secantes en los puntos A, A0 , B,
AB A0 B 0
B 0 , C, C 0 respectivamente, tales que AA0 k CC 0 y = 0 0 . Por el punto B tracemos una
BC BC
recta paralela a AA0 , la cual interseca a A0 C 0 en el punto D (ver figura 70). Entonces, por el
AB A0 D A0 B 0 A0 D
Teorema de Thales se tiene que: = . De donde, 0 0 = , as por las propiedades
0 0 0 0
BC DC 0 BC DC 0
AC AC
de las proporciones 0 0 = , por lo que B 0 C 0 = B 0 D + DC 0 = DC 0 y por tanto B 0 D = 0,
B0 C DC 0
o equivalentemente B = D y por lo tanto, BB 0 k AA0 .

Figura 70: Recproco del Teorema de Thales

Corolario (Recproco del Teorema de Thales en el triangulo.) Si una recta intercepta


dos lados de un triangulo en segmentos proporcionales entonces la recta es paralela al tercer
lado del triangulo.

Triangulos semejantes. Decimos que el 4ABC es semejante al 4A0 B 0 C 0 (Ver figura 71), lo
cual denotamos as ABC A0 B 0 C 0 , si:
AB AC BC
= =
A0 B 0 A0 C 0 B0C 0
y
BAC = B 0 A0 C 0 , ABC = A0 B 0 C 0 , ACB = A0 C 0 B 0 .

En los tres teoremas que se muestran a continuacion (los cuales son una consecuencia directa del
Teorema de Thales) se establecen las condiciones mnimas para demostrar que dos triangulos
son semejantes, a los cuales denominaremos: Criterios de Semejanza de Triangulos.

Primer criterio de semejanza de triangulos: Angulo-Angulo A-A. Si dos angulos de


un triangulo son congruentes con dos angulos de otro triangulo, entonces los dos triangulos son
semejantes.

48
Figura 71: Definicion de Semajanza de Triangulos.

Demostracion. Supongamos que en el 4ABC y 4A0 B 0 C 0 se tiene que ABC = A0 B 0 C 0


y ACB = A0 C 0 B 0 , entonces BAC = B 0 A0 C 0 (Por la suma de angulos internos en un
triangulo).
Sea D AB y E AC tales que AD = A0 B 0 y AE = A0 C 0 , dado que DAE = BAC =
B 0 A0 C 0 , se sigue por L-A-L que 4ADE = 4A0 B 0 C 0 , por consiguiente ADE = A0 B 0 C 0 =
ABC, de donde DE k BC (por ser iguales los angulos correspondientes) y por el teorema de
Thales
AB AC
=
AD AE
y por consiguiente
AB AC
0 0
= 0 0 (1)
AB AC
Sea F BC tal que DF k AC, entonces F C = DE = B 0 C 0 (porque DECF es paralelogramo
y por ser 4ADE = 4A0 B 0 C 0 ) y por el teorema de Thales

BA BC
=
DA FC
o lo que es lo mismo
AB BC
0 0
= 0 0 (2)
AB BC
Luego, de (1) y (2) se tiene que:

AB AC BC
= = .
A0 B 0 A0 C 0 B0C 0
As, se ha demostrado que los tres pares de angulos son congruentes y los tres pares de lados
son proporcionales, por lo tanto, 4ABC 4A0 B 0 C 0 .

Segundo criterio de semejanza de triangulos: L-A-L. Si un angulo de un triangulo es


congruente con otro angulo de otro triangulo y los lados que comprenden al angulo en el primer
triangulo son respectivamente proporcionales a los lados que comprende al angulo en el segundo
triangulo, entonces los dos triangulos son semejantes.
AB AC
Demostracion. Suponga que el BAC = B 0 A0 C 0 y que 0 0 = 0 0 . Considere los pun-
AB AC
tos D y E, como en la demostracion del teorema anterior. Entonces por el criterio L-A-L,

49
4ADE = 4A0 B 0 C 0 , de lo cual se deduce que ADE = A0 B 0 C 0 . Por otra parte tene-
AB AC
mos que: = , y al aplicar el recproco del teorema de Thales, se puede afirmar que
AD AE
DE k BC, de lo cual a su vez se deduce que ADE = ABC, por angulos correspondientes
entre paralelas. Finalmente por transitividad se concluye que ABC = A0 B 0 C 0 . Por lo tanto,
4ABC 4A0 B 0 C 0 (Por el criterio A-A.)

Tercer criterio de semejanza de triangulos: L-L-L. Si los tres lados de un triangulo son
respectivamente proporcionales a los tres lados de otro triangulo, entonces los dos triangulos
son semejantes.
AB AC BC
Demostracion. Por hipotesis se tiene que: 0 0 = 0 0 = 0 0 y como antes sean D y E
AB AC BC
puntos sobre AB y AC respectivamente tales que AD = A0 B 0 y AE = A0 C 0 . Entonces por
el recproco del teorema de Thales se tiene que DE k BC y por consiguiente el ABC =
ADE y el ACB = AED, de donde 4ABC 4ADE (por el criterio A-A). Por ende
AB BC BC BC
= , luego por transitividad = 0 0 , de donde DE = B 0 C 0 . En consecuencia
AD DE DE BC
4ADE = 4A0 B 0 C 0 (por el criterio L-L-L), de lo cual se sigue que A0 B 0 C 0 = ADE y
A0 C 0 B 0 = AED, y por transitividad A0 B 0 C 0 = ABC y A0 C 0 B 0 = ACB =. Por lo
tanto, 4A0 B 0 C 0 4ABC (Por el criterio A-A.)

Ejercicios
1. Sean AB y CD las bases del trapecio ABCD, cuyas diagonales son perpendiculares. Si
se sabe que AD = 13, AE = 12 y CE = 4 encuentre las longitudes de CD y AB.

2. En la figura 72, el 4ABC es equilatero, sus lados tienen longitud 3 y P A es paralela a


BC. Si P Q = QR = RS, encontrar la longitud de CS.

Figura 72

3. Sea ABCD un trapecio de bases BC y AD, sus diagonales se cortan en E. Si BE = 3,


ED = 4 y CE = 2, determine la medida de AE.

4. Las bases de un trapecio miden 3 y 5, y si su altura mide 4. Encontrar la distancia desde


el punto de corte de las diagonales hasta la base mayor.

50
5. En la figura adjunta, el 4ABC es rectangulo en A y el 4ADB
es rectangulo en D. El punto E es el punto de interseccion de
los segmentos AD y BC. Si AC = 15, AD = 16 y BD = 12,
calcule el area del 4ABE.

6. El 4ABC es rectangulo en B. Se dibuja un rectangulo BEDF con D sobre la hipotenusa,


BC 1
E y F sobre BC y AB, respectivamente. Si AB = 1, demuestre que BE = 1DE .

7. Considerese los puntos A, B, C y D tales que A y B estan sobre el segmento OC y


OD respectivamente, donde O es el centro de la circunferencia de radio r (Ver figura
73). Si OA OC = r2 = OB OD, demuestre que el 4AOB ' 4DOC y que CD =
r2
 
AB.18
OA OB

Figura 73

8. Sobre la circunferencia de centro O, se trazan los diametros AB y CD tales que AB CD.


Sea P un punto sobre el arco CBD y Q el punto de interseccion de las cuerdas AP y
CD. Si DO = 1, demuestre que AP AQ = 2.

9. Un segmento de recta AB es divido por los puntos interiores K y L de manera que


AL2 = AK AB. Sea P un punto exterior al segmento AB tal que AP = AL. Pruebe que
KP L = LP B. Figura 74.

Figura 74

18
La medida del segmento CD se denomina Distancia Inversa.

51
10. En la figura 75, AB y AC son tangentes a la circunferencia, y CE BD, siendo BD un
diametro. Probar que BE BO = AB CE.

Figura 75

1 1 1
11. Demostrar que + = si se cumple que AX k BY k CZ. (Ver figura 76.)
AX BY AZ

Figura 76

12. En la figura 77, el 4ABC es rectangulo. Se construyen exteriormente los cuadrados


ABEF y BCP Q. Demostrar que BM = BN .

Figura 77: .

52
13. Sean O, P y R los centros de las tres circunferencias. Si OR = r y Q es la interseccion
de P O con la circunferencia de centro R, demuestre que OP OQ = r2 . Ver figura 78.

Figura 78

14. Si en un triangulo rectangulo se traza la altura correspondiente a la hipotenusa, entonces:

a) Los dos nuevos triangulos que resultan, son semejantes entre si y semejantes al
triangulo original.
19
b) La altura es media proporcional entre los segmentos que ella determina sobre la
hipotenusa.
c) Cada cateto es media proporcional entre la hipotenusa y la proyeccion del cateto
sobre la hipotenusa.
d ) Demuestre el teorema de Pitagoras.

15. Si dos triangulos tienen sus lados respectivamente paralelos o respectivamente perpendi-
culares, entonces los dos triangulos son semejantes.

16. Las alturas, las bisectrices y las medianas homologas de dos triangulos semejantes estan
en la misma razon que sus lados homologos.

17. Sean ABC y A0 B 0 C 0 dos triangulos semejantes con AAB BC CA


0 B 0 = B 0 C 0 = C 0 A0 = k. Demuestre

que: la razon entre los permetros de los triangulos es k y que la razon entre sus areas es
k2.

18. Teorema de Menelao. Dado el 4ABC, sea P un punto sobre la recta AB, Q un punto
sobre la recta BC, R un punto sobre la recta CA. Si los puntos P , Q, R estan alineados
AP BQ CR
entonces = 1.
P B QC RA
Para demostrar este teorema, sea W un punto sobre la recta P QR tal que BW k AC:

a) Demuestre que los triangulos AP R y BP W son semejantes.


19
Si b es una magnitud tal que ab = cb , entonces decimos que b es media proporcional entre a y c,o de manera
equivalente: b es media proporcional entre a y c si y solo si b2 = a c.

53
Figura 79: Teorema de Menelao.

b) Demuestre que los triangulos CQR y BQW son semejantes.


AP BQ CR
c) De los literales a) y b) deduzca que = 1.
P B QC RA

19. Teorema de Ceva. Dado el 4ABC, sea P un punto sobre el


recta AB, Q un punto sobre la recta BC y R un punto sobre
la recta CA. Si las rectas AQ, CP , BR concurren, entonces
AP BQ CR
= 1.
P B QC RA

Para demostrar este teorema, sean W y V los puntos de interseccion de la recta que pasa
por B paralela a AC, con las rectas CP y AQ, respectivamente.

a) Demuestre que 4AP C 4BP W y que 4AQC 4V QB.


b) Demuestre que 4BW P 4RCP y que 4BV P 4RAP .
AP BQ CR
c) Utilice los literales a) y b) para probar que = 1.
P B QC RA
20. Si dos cuerdas se interceptan en el interior de una circunferencia entonces el producto
de las medidas de los segmentos determinados por el punto de interseccion en una de las
cuerdas es igual al producto de las medidas de los segmentos determinados en la otra
cuerda.

21. Si dos segmentos se interceptan en un punto que esta en el interior de los dos segmentos
y el producto de las medidas de los segmentos determinados por el punto de interseccion
en el primer segmento es igual al producto de las medidas de los segmentos determinados
por el punto en el segundo segmento,entonces los extremos de los segmentos estan sobre
una circunferencia.

22. Si desde un punto P exterior a una circunferencia se trazan dos semirrectas secantes
que cortan a la circunferencia en los puntos A, B y C, D respectivamente, entonces
P A P B = P C P D.

54
23. Si desde un punto P se trazan dos semirrectas con los puntos A, B sobre una y los puntos
C, D sobre la otra, tales que P A P B = P C P D, entonces los puntos A, B, C, D estan
sobre una circunferencia.

24. Si desde un punto exterior a una circunferencia se trazan dos semirrectas, una tangente
y la otra secante, entonces el segmento entre el punto y el punto de tangencia es media
proporcional entre los segmentos determinados entre el punto exterior y los puntos de
interseccion de la secante con la circunferencia. 20

25. Si P es un punto sobre el mismo plano que una circunferencia de centro O y radio r, y d
es la distancia del punto P al centro O de la circunferencia, demuestre que:

a) Si P esta en el interior de la circunferencia, entonces la potencia de P es r2 d2 .


b) Si P esta en el exterior de la circunferencia, entonces la potencia de P es d2 r2 .
c) Si P esta sobre de la circunferencia, entonces la potencia de P es cero.

26. (*) (IV OMCC, P-4) Sea ABC un triangulo, D el punto medio de BC, E un punto sobre
el segmento AC tal que BE = 2AD y F el punto de interseccion de AD con BE. Si
CAD = 60, encuentre la medida de los angulos del 4F EA.

27. (*) Sea ABCD es un trapecio con AD k BC. M y N son los puntos medios de CD y BC,
respectivamente, y P el punto comun de las rectas AM y DN . Si PAP
M
= 14 , demuestre que
ABCD es paralelogramo.

28. Dado el 4ABC se construye un cuadrado P QRS con P en AB, Q en AC, R y S en BC.
Sea H el pie de la altura desde A hacia BC. Demuestre que:
1 1 1
a) = +
PQ AH BC
b) (ABC) = 2(P QRS) si y solo si AH = BC.

29. Sea P un punto en el interior del 4ABC. Se trazan por P las paralelas a los lados del
triangulo, que queda dividido en tres triangulos y tres paralelogramos. Si las areas de los
tres triangulos de la subdivision son, en algun orden, 9, 16 y 25, hallar el area del 4ABC.

Problemas de Refuerzo.

30. (*) En la figura anexa, BC = CD = DE =


EA = x y AEB = 90. Demuestre que
ABC + ACD + ADE = 90.

20
Los problemas anteriores nos permite establecer la siguiente definicion de Potencia de un punto con
respecto a una circunferencia: La potencia de un punto P con respecto a una circunferencia de centro O y
radio r es el producto P A P B, donde A y B son los puntos de interseccion de la circunferencia con una recta
que pasa por P .

55
31. Las tres circunferencias de la figura 80 tienen el mismo radio r, sus centros son colineales
y la circunferencia de centro O2 es tangente a las otras dos. Por A se traza una tangente
a la circunferencia de centro O3 . Obtenga el valor del segmento BC en funcion de r.

Figura 80

32. Sea ABCD un rombo, con AC = 6 y BD = 8. Se construyen exteriormente los cuadrados


ADEF y CDHG, cuyos centros son O1 y O2 , respectivamente (Vea figura 81). Calcular
la medida del segmento O1 O2 .

Figura 81

33. Sea ABCD un cuadrado con P y Q sobre AB y BC tales que BP = BQ. Sea H el pie
de la perpendicular de B a P C. Demuestre que DHQ = 90.

34. Dado un paralelogramo ABCD, se trazan dos circunferencias tangentes externamente


en P , y tales que la primera es tangente internamente al ABC y la otra es tangente
internamente al CDA, como en la figura 82. Demuestre que B, P y D estan alineados.

56
Figura 82

35. En un 4ABC el CAB = 120. Encuentre la medida de la bisectriz interna del CAB
en funcion de los lados adyacentes.

36. El 4ABC tiene lados de 13, 14 y 15 unidades. El 4A0 B 0 C 0 esta dentro del 4ABC con
lados paralelos a los de este y a 2 unidades de distancia de los lados del mismo. Calcule
(ABC) (A0 B 0 C 0 ).

37. (*) (Asiatico Pacfica) Sea ABC un triangulo y D el pie de la altura con respecto a A.
Sean E y F puntos en una recta que pasa por D (distintos de D) tales que AE CE
y AF BF . Sean M y N los puntos medios de BC y EF , respectivamente. Demuestre
que AN N M .

57
7. Puntos y Rectas Notables del Triangulo.
MEDIANAS
Definicion: En un triangulo, una mediana es el segmento de recta que une un vertice con el
punto medio del lado opuesto.

Teorema: Las tres medianas de un triangulo concurren en un punto llamado el Centroide 21


del triangulo y usualmente es denotado por G. Ademas, las medianas de cortan mutuamente
en razon 2:1.

Demostracion: Dado el 4ABC sean A0 , B 0 , C 0 , los puntos medios de BC, CA, AB, respec-
tivamente. Defina G como la interseccion de BB 0 con CC 0 . Por el teorema de la base media,
B 0 C 0 k BC y 2B 0 C 0 = BC; observe que 4BCG ' B 0 C 0 G, con razon de semejanza 2, por lo
que
GB GC
0
= =2
GB GC 0
Analogamente, si G = AA0 BB 0 se cumple
G B G A
= =2
G B 0 G A0
As, G y G dividen al segmento BB 0 en dos segmentos cuya razon es 2:1, por lo que G = G ,
lo cual implica que AA0 , BB 0 , CC 0 concurren y
GA GB GC
0
= 0
= =2
GA GB GC 0

MEDIATRICES
Definicion: Dado un segmento AB, la mediatriz del segmento es el lugar geometrico de puntos
que equidistan de A y B, i.e., un punto P esta sobre la mediatriz de AB si y solo si P A = P B.

Teorema: La mediatriz de AB es una recta l perpendicular a AB y que pasa por su punto


medio.

Demostracion: Sea M el punto medio de AB, y l pasa por M y l AB. En primer lugar
se probara que todos los puntos de l satisfacen la definicion de mediatriz: Por definicion de
punto medio M A = M B. por lo que claramente M pertenece a la mediatriz de AB; sea P un
punto de l distinto de M , por criterio LAL, 4P M A 4P M B por lo que P A = P B. Ahora,
cabe preguntarse si existe algun punto fuera de l que tambien cumpla la definicion: suponga
P 0 tal que P 0 A = P 0 B, esto implica que 4P 0 AB es isosceles, y entonces P 0 AB = P 0 BA;
si M 0 es la proyeccion de P 0 sobre AB, por criterio ALA 4P 0 AM 0 4P 0 BM 0 , lo cual impli-
ca que M 0 A = M 0 B, es decir que M 0 = M , y esto obliga a que P 0 este sobre l (ya que P 0 M 0 = l).

Teorema: Las mediatrices de un 4ABC concurren en un punto que equidista de los vertices
del triangulo, llamado el Circuncentro del 4ABC
21
Tambien conocido como Geocentro, Centro de Gravedad, Baricentro, o mas formalmente Equibaricentro.

58
Figura 83: Concurrencia de Mediatrices, Circuncentro y Circuncrculo.

Usualmente, el circuncentro es denotado por O, y R representa la distancia del circuncentro a


los vertices
R = OA = OB = OC
A esta distancia se le llama Circunradio del 4ABC. As, O es el centro de una circunferencia
que pasa por A, B, C, cuyo radio es R, llamada el Circuncrculo del 4ABC.22

Demostracion: Sea O la interseccion de las mediatrices de AB y BC, por el teorema anterior,


como O pertenece a la mediatriz de AB se cumple OA = OB, y como tambien pertenece a la
mediatriz de BC, OB = OC; entonces OC = OA, y utilizando de nuevo el teorema anterior,
O debe pertenecer a la mediatriz de CA. As, las tres mediatrices concurren en O, y este punto
equidista de los vertices del 4ABC.

Corolario: Dado un triangulo, existe una circunferencia que pasa por los tres vertices (el cir-
cuncrculo); ademas, esta circunferencia es unica.

Una observacion importante es que la mediatriz del lado de un triangulo NO siempre pasa por
el vertice opuesto; de hecho, esto solo se da si el triangulo es isosceles.

ALTURAS
La altura es un concepto que esta intrnsecamente relacionado con la distancia de un punto a
una recta; la altura es la recta que debe trazarse para determinar esta distancia, i.e., es una
recta que pasa por el punto y es perpendicular a la recta. A la interseccion entre la altura y la
recta generalmente se le llama pie de la altura, o tambien (mas formal) proyeccion del punto
sobre la recta. En particular, para triangulos, definiremos la altura de la siguiente forma:

Definicion: Dado un triangulo, una altura es una recta que pasa por un vertice y es perpen-
dicular al lado opuesto.

22
En ocasiones, denotaremos a esta circunferencia por (ABC).

59
Es importante observar que el pie de la altura NO siempre pertenece a un lado; de hecho, una
altura puede estar al interior de un triangulo, coincidir con un lado, o estar completamente
afuera de un triangulo.

Teorema: Las alturas de un triangulo concurren en un punto, llamado el Ortocentro del triangu-
lo, usualmente denotado por H.23

Demostracion: Dado el 4ABC, se construyen los puntos A1 , B1 , C1 , tales que ABA1 C,


BCB1 A, CAC1 B son paralelogramos. Observe que el 4ABC es el triangulo medial del 4A1 B1 C1 ,
y que las alturas del 4ABC son las mediatrices del 4A1 B1 C1 ; como las mediatrices de un
triangulo concurren (en este caso, las del 4A1 B1 C1 ), las alturas del 4ABC concurren.

La altura tambien puede escribirse en terminos de lugar geometrico:

Teorema: La recta l es perpendicular a AB si y solo si AL2 LB 2 es constante. Es decir, que


una recta perpendicular a AB es el lugar geometrico de los puntos L que satisfacen la condicion
anterior.

Demostracion: sea P la interseccion de l con AB, y L un punto arbitrario sobre l; por Pitago-
ras se tiene AL2 LB 2 = AP 2 P B 2 , y el termino derecho de la igualdad es constante. La
otra direccion de la implicacion se prueba por contradiccion.

De esa definicion tambien puede fabricarsele una demostracion del teorema anterior, sin em-
bargo, no se aborda porque la prueba se basa en un resultado sofisticado llamadado Teorema
de Steiner.24

BISECTRICES
Definicion: La bisectriz de un angulo es una recta que divide al angulo en dos angulos de
igual magnitud.

Teorema: El lugar geometrico de puntos que equidistan de dos rectas dadas, generan un par
de rectas perpendiculares llamadas bisectriz interna y bisectriz externa del angulo formado por
las rectas.

Demostracion: Suponga que las rectas se cortan en un punto O; sean a, b las rectas dadas, y
P un punto que equidista de ellas; si A y B son las proyecciones de P sobre a y b, respectiva-
mente, entonces P A = P B. Observe que por criterio LLL (utilizando Pitagoras previamente),
4OAP 4OBP , por lo que P OA = P OB, i.e., P pertenece a la bisectriz del AOB.
Claramente aqu se dan dos casos, recuerde que para definir el angulo entre a y b se utilizan uni-
camente semi-rectas, por lo que las rectas a y b definen cuatro angulos, que por parejas pueden
23
El triangulo formado por los pies de las alturas de un 4ABC es llamado el triangulo ortico del 4ABC.
24
Sean l, m, n, tres rectas perpendiculares a los lados del AB, BC, CA del 4ABC, respectivamente. Sean
L, M , N , puntos arbitrarios sobre l, m, n, respectivamente. Entonces las rectas l, m, n concurren si y solo si
AL2 + BM 2 + CN 2 = N A2 + LB 2 + M C 2 .

60
ser opuestos por el vertice o suplementarios; de estos se escoge cualquiera de ellos como referen-
cia, entonces, si AOB coincide con este o con el opuesto por el vertice, la recta P O es llamada
bisectriz interna, y en caso contrario, bisectriz externa. As, el lugar geometrico son dos rectas,
y su perpendicularidad se basa en los pares de angulos que son suplementarios. Finalmente, si
a k b, el lugar geometrico es una recta paralela a a y b que se ubica entre ellas a igual distancia de
ambas (este es un caso extrano de bisectriz interna, sin embargo, en ocasiones es util tener esta
convencion en mente; peor aun, la bisectriz externa es una recta ideal llamada recta al infinito).

Teorema: Las bisectrices internas de un 4ABC concurren en un punto, llamado el Incentro


del 4ABC, usualmente denotado por I. La distancia de I a los tres lados del triangulo es igual
a un numero r, llamado el Inradio del 4ABC, y de aqu que la circunferencia de centro I y
radio r sea tangente a los lados del triangulo; dicha circunferencia es llamada el Incrculo del
4ABC.25

Figura 84: Concurrencia de Bisectrices Internas, Incentro e Incrculo.

Demostracion Sea I la interseccion de las bisectrices internas de A y B (obviamente, I


esta en el interior del 4ABC); como I pertenece a la bisectriz interna del A, por el teorema
anterior dist(I, AB) = dist(I, AC), y analogamente, como I pertenece a la bisectriz interna del
B, dist(I, AB) = dist(I, CB); entonces dist(I, AC) = dist(I, CB), y de nuevo por el teorema
anterior y dado que I esta al interior del triangulo, I pertenece a la bisectriz interna del C.
As, las tres bisectrices internas concurren en un punto que equidista de los lados del triangulo.

Es importante notar que las interseccion de una bisectrices interna con el lado opueto del
triangulo NO siempre coincide con el puntos de tangencia del incrculo;26 de hecho, esto ocurre
solamente si el triangulo es isosceles.

Corolario: Dado un triangulo, existe una circunferencia que es tangente interiormente a los
tres lados (el incrculo); ademas, esta circunferencia es unica.27
25
En algunas ocasiones denotaremos al incrculo por (ABC).
26
En la figura, el 4ABC es llamado triangulo tangencial del 4DEF .
27
Existen 3 circunferencias mas que son tangentes a los tres lados del triangulo, llamados excrculos; estas
circunferencias se ubican en el exterior del triangulo.

61
Ejercicios
1. Las areas de los seis triangulos AGB 0 , AGC 0 , BGA0 , BGC 0 , CGA0 , CGB 0 son iguales e
iguales a un 61 del area del triangulo ABC.

Figura 85

2. Los cuatro triangulos AB 0 C 0 , BC 0 A0 , CA0 B 0 , A0 B 0 C 0 ,28 son congruentes entre si y seme-


jantes al 4ABC con razon de semejanza 12 .

3. El centroide del 4ABC coincide con el centroide del triangulo medial 4A0 B 0 C 0 . Ademas,
estos dos triangulos tienen lados correspondientes paralelos (triangulos homoteticos).

4. En la figura 86, G es el centroide. Si GD = 2 y el area sombreada vale 5, calcule AD y el


(ABC).

Figura 86

5. Demostrar que las paralelas a los lados de un 4ABC, trazadas por el centroide G dividen
cada lado en tres partes iguales.

6. ABCD es un paralelogramo de centroide (baricentro) E, M es el punto medio de AD,


y F es la interseccion de AC con BM . Si el area de ABCD es 1, calcule el area del
cuadrilatero DEF M .

7. En el 4ABC, se traza la mediana AM . Demostrar que si BM = AM , entonces el


triangulo es rectangulo en A.
28
El 4A0 B 0 C 0 es llamado el triangulo medial del 4ABC.

62
8. La suma de las distancias del centroide a los puntos medios de los lados de un triangulo
es 20. Calcule la suma de las medianas del triangulo.

9. La mediana tiene longitud menor que la semisuma de los lados adyacentes, es decir AA0 <
b + c, BB 0 < c + a, CC 0 < a + b.

10. Dado el 4ABC, sean D y E puntos variables sobre los lados AB y AC respectivamente
tales que BC k DE. Entonces, la mediana AA0 puede definirse como el lugar geometrico
de los puntos P tales que P CD BE.29

11. Siempre es posible construir un triangulo XY Z con las medianas AA0 , BB 0 , CC 0 de un


4ABC dado. Ademas, los segmentos que unen el centroide del 4XY Z con sus vertices
son iguales a la mitad de los lados del 4ABC.

12. En el 4ABC, AB = BC y la mediatriz de BC interseca a la mediana BM en L. Si


LCB = 25, determine la medida del LAC.

13. Ley del Seno. Dado un 4ABC, se cumple que


senA senB senC 1
= = =
a b c 2R

14. Las reflexiones de H con respecto a los lados del 4ABC caen sobre el circuncrculo del
mismo, es decir HHa = Ha X y analogo para los otros lados.

Figura 87

15. Las reflexiones de H con respecto a los puntos medios de los lados del triangulo, caen
sobre el circuncrculo del mismo.
29
Si D = A se define P = A, y cuando D = B entonces P es punto medio de BC.

63
16. Si O y H son el circuncentro y el ortocentro de un 4ABC, respectivamente, entonces
BAH = CAO.

17. La altura AHa es bisectriz del Hb Ha Hc .

18. Los circuncrculos de 4ABC, 4ABH, 4BCH, 4CAH tienen igual radio.

19. La perpendicular trazada desde A al lado Hb Hc del triangulo ortico, pasa por el circun-
centro del 4ABC.

20. A, B, C y H forman un cuadrilatero ortocentrico, es decir que cada punto es el ortocentro


del triangulo formado por los otros tres.

21. El ortocentro de un triangulo esta al interior, sobre un vertice, o afuera del triangulo, si
el triangulo es acutangulo, rectangulo, u obtusangulo, respectivamente.

22. El circuncentro del 4ABC es el ortocentro del triangulo medial 4A0 B 0 C 0 .

23. Sea O el circuncentro del 4ABC. Si AOC = 100 y OCB = 30, determine la medida
de los angulos del 4ABC.

24. Hallar los angulos de un triangulo cuyo triangulo ortico tiene angulos de 20, 50 y 110.

25. Sea ABC un triangulo obtusangulo de circuncentro O y altura AD. Si OAB = 25 y


OCB = 15, calcule el DAB.

26. El 4ABC de circuncentro O y altura BD. Si DAB = 35 y OBD = 10 encontrar los


angulos del triangulo ABC.

27. En la figura 88, AB es diametro de la circunferencia. Si X es la interseccion de CG con


AB, calcular el CXB.

Figura 88

64
28. En el 4ABC, se trazan la altura AH y la mediana BM . Demuestre que el 4M HC es
isosceles.
AB
29. Un 4ABC es rectangulo en C, A = 75 y CH es altura. Demuestre que CH = .
4
30. Sea O el circuncentro del 4ABC con C = 45 y sea D el pie de la altura desde A.
Calcule la medida del ODC.

31. Dado el 4ABC isosceles con A = 90, sean P y Q son puntos dentro del triangulo tales
que BP = AQ y AP = CQ. Si BP y CQ se cortan en R, demostrar que AR P Q.

32. Se ubican los puntos M y K sobre los lados BC y CD del cuadrado ABCD, respecti-
vamente, de modo que M C = KD. Sea P la interseccion de M D y BK, demuestre que
AP M K.

33. Sean D, E, F los puntos de tangencia del incrculo sobre los lados BC, CA, AB del
4ABC. Demuestre que se cumplen las siguientes relaciones, donde s denota el semi-
permetro del triangulo:
AE = AF = s a
BD = BF = s b
CD = CE = s c

34. El ortocentro del 4ABC es el incentro de su triangulo ortico.

35. Dado un 4ABC, su triangulo ortico y su triangulo tangencial tienen lados correspon-
dientes paralelos (triangulos homoteticos).

36. Las bisectrices exteriores de B y C, junto con la bisectriz interior de A, concurren


en un punto, llamado el Excentro con respecto al vertice A, usualmente denotado por Ia .
Este punto es equidistante a los lados del 4ABC, dicha distancia es el Exradio respecto a
A, usualmente denotado por ra . As, la circunferencia de centro Ia y radio ra es tangente
exteriormente a los lados del 4ABC, y es llamada el Excrculo respecto a A.30

37. I es ortocentro del 4Ia Ib Ic . Ademas se cumple:

AX = AZ = s
BX = BY = s b
CY = CZ = s c

38. En un 4ABC, la bisectriz exterior del ABC y la bisectriz exterior del BCA se cortan
en D. La paralela a BC por D corta a AC en L y a AB en M . Si LC = 5 y M B = 7,
hallar LM .

39. El 4ABC es rectangulo en A. Si I es el incentro, calcular BIC.

40. En un 4ABC, el ABC CAB = 90. Sean D y E los pies de las bisectrices interior
y exterior del BCA respectivamente. Demuestre que CD = CE.
30
Analogamente se definene los excrculos con respecto a los otros vertices.

65
Figura 89

41. En el 4ABC, AB < AC, AD es bisectriz, y E es un punto en AB tal que el EDB = 90.
El punto F sobre AC es tal que el BED = DEF . Demuestre que el BAD = F DC.
42. En el 4ABC se trazan las bisectrices interiores BD y CE tales que D es el punto sobre
AC, E es el punto sobre AB, 2BDE = 3B y CED = 2B. Calcular los angulos del
4ABC.
43. Dado el 4ABC con A = 90, sea D el pie de la perpendicular desde A. Sean ademas I
y J los incentros respectivos de 4ABD y 4ACD. Demostrar que la bisectriz del BAC
es perpendicular a IJ.
44. Un triangulo es isosceles si cumple alguna de las siguientes condiciones:
a) Dos medianas son iguales.
b) Dos alturas son iguales.
c) Dos bisectrices son iguales.31
45. Teorema de la Bisectriz: Dado el 4ABC, sean P y P 0 sobre BC. Se cumple que AP
y AP 0 son la bisectriz interna y la bisectriz externa del A si y solo si
BP BA BP 0
= = 0
PC AC PC
Sugerencia: Para demostrar la primera igualdad, trace CD k AP con E sobre la prolon-
gacion de AB.
31
Este caso es aparentemente tan sencillo como los anteriores, pero realmente es un resultado muy complicado
y recibe el nombre de Teorema de Steiner-Lehmus.

66
46. (*) De acuerdo con los datos de la grafica 90, calcular el valor de AB.

Figura 90

47. Dos circunferencias son tangentes internamente en P , y una cuerda AB de la circunfe-


rencia de radio mayor es tangente en Q a la otra circunferencia. Ver figura 91.

a) Demuestre que P Q es bisectriz del AP B.


b) Llame A0 y B 0 a las otras intersecciones de P A y P B con la circunferencia de radio
menor y suponga que AB = 15, P A0 = 3 y P B 0 = 2; calcule AQ y BQ.

Figura 91

48. (*) Sea ABC un triangulo tal que las medianas respectivas a B y C son perpendiculares.
Demuestre que se cumple la relacion.

5BC 2 = CA2 + AB 2 .

49. (*) Teorema de Poncelet: Demuestre si 4ABC es un triangulo rectangulo con A =


90, entonces 2(r + R) = b + c.

67
Problemas de Refuerzo.

50. (*) Sea ABCD un paralelogramo. Q es el punto medio de AD, F el pie de la perpendicular
por B sobre QC. Probar que AF = AB.

51. Dado el rombo ABCD, se trazan las bisectrices internas de DAC, CAB, BCA,
ACD, y cortan a DC, CB, BA, AD en P , Q, R, S, respectivamente. Demuestre que
P QRS es un rectangulo.

52. (*) Sea ABCD un cuadrilatero tal que AB = CD. Las mediatrices de AC y BD se cortan
en P . Probar que P AC = P CA = P BD = P DB.

53. (*) ABC es un triangulo y P un punto en su interior. Sean A0 , B 0 y C 0 las reflexiones de


P sobre BC, CA y AB, respectivamente. D, E y F son los pies de las perpendiculares
respectivos desde A, B y C hacia B 0 C 0 , C 0 A0 y A0 B 0 . Probar que AD, BE y CF son
concurrentes.

54. (*) (Arnoldo Aguilar) En la figura 92, ABGH, BCF G y CDEF son cuadrados. Si I es
el centro de ABGH y J = DH BG, demuestre que I, J y F estan alineados.

Figura 92

55. (*) (Arnoldo Aguilar) Sea ABC un triangulo equilatero. M y N son los puntos medios de
AB y BC, respectivamente. Exteriormente al 4ABC se construye un triangulo rectangulo
isosceles 4AP C, con AP C = 90 . Si I es la interseccion de AN y M P , demuestre que
CI es la bisectriz de ACM .

56. (*) En la figura adjunta, el 4ABC es tal que A = 90 y


B = 60. Cual es el radio de la circunferencia?

57. (*) Dado el paralelogramo ABCD, sea M el punto medio de AB, y N la interseccion de
CD con la bisectriz interna del ABC. Demuestre que M C BN si y solo si AN es
bisectriz del DAB.

68
58. (*) En el 4ABC, se sabe que los vertices B, C, el circuncentro O y el ortocentro H del
4ABC estan todos sobre una misma circunferencia.

a) Calcule el valor de A.

b) Demuestre que el incentro tambien pertenece al circuncrculo de BCOH.

59. (*) Sea ABC un triangulo de ortocentro H. Sean P y Q los pies de las perpendiculares
desde H a las bisectrices interior y exterior de A, respectivamente. Si M es el punto medio
de BC, mostrar que P , Q y M estan alineados.

60. (*) En un triangulo ABC, sea M el punto medio de BC. Si se cumple que AB 6= AC y
ademas M AC + ABC = 90 , hallar BAC.

61. (*) Sea ABC un triangulo y U un punto de su circuncrculo tal que AU es bisectriz. Las
mediatrices en AB y AC cortan a AU en X y Y . Sea T la interseccion de BX con CY .
Demostrar que AU = T B + T C.

62. (*) (The 59th Romanian Mathematical Olympiad Final Round) Sea ABCD un rectangulo
de centro O con AB 6= BC. La perpendicular en O a BD corta a las lneas AB y BC en
los puntos E y F , respectivamente. Sean M y N los puntos medios de los segmentos CD
y DA, respectivamente. Probar que las lneas rectas F M EN .

63. (*) Sea ABC un triangulo rectangulo, con A = 90 . Sea D un punto en su interior tal
que DAC = DCA = DBC = , y AC = BD. Determine el valor de .

64. (*) Sea ABC un triangulo y M un punto tal que M AB = 10, M BA = 20, M AC =
40 y M CA = 30. Probar que el 4ABC es isosceles.

65. (*) En la figura 93, ABCD y P QRS son cuadrados, 4ABP 4BCQ 4CDR
4DAS y los los radios de las cinco circunferencias son iguales a r. Si a es el lado del
cuadrado ABCD, determine r en funcion de a.

Figura 93

69
66. (*) Recta de Euler. El centroide G, el ortocentro H y el circuncentro O de un triangulo
estan alineados, y ademas GH = 2GO.

67. Circunferencia de los 9 puntos:32 Dado un 4ABC de ortocentro H, se cumple que


los puntos medios de los lados, los pies de las alturas, y los puntos medios de HA, HB,
HC, se ubican sobre una misma circunferencia. Ademas, el centro de esta circunferencia
es el punto medio de HO y su radio es R2 , donde O y R son el circuncentro y el circunradio
del triangulo.
Para demostrar este resultado se sugiere seguir los siguientes pasos:

a) Si Ha es el pie de la altura trazada desde A, demuestre que la reflexion de H con


respecto a Ha pertenece a circuncrculo del 4ABC. Resultados similares se cumplen
para Hb y Hc .
b) Si A0 es el punto medio de BC, demuestre que la reflexion de H con respecto a A0
pertenece al circuncrculo del 4ABC.
c) De los resultados anteriores, observe que hay 9 puntos sobre el circuncrculo del
4ABC: los vertices, las reflexiones de H con respecto a los pies de las alturas, y
las reflexiones de H con respecto a los puntos medios de los lados; a partir de esto,
concluya que los puntos medios de los segmentos que van de H a estos 9 puntos,
tambien deben pertenecer en una misma circunferencia.
d) Concluya ademas que el centro de esta nueva circunferencia es el punto medio de HO.

Otro camino de solucion es el siguiente:

a) Sea 4A0 B 0 C 0 el triangulo medial del 4ABC. Pruebe que A0 B 0 C 0 = BHa C 0 y


concluya que Ha A0 B 0 C 0 es un cuadrilatero cclico; los mismo debe cumplirse para Hb
y Hc .
b) Sea X el punto medio de HA. Demuestre que B 0 A0 C 0 +B 0 XC 0 = 180 y concluya que
XC 0 B 0 A0 es un cuadrilatero cclico; lo mismo debe cumplirse para los puntos medios
de HB y HC.
c) De lo anterior, concluya que los pies de las alturas y los puntos medios de los segmentos
que van desde H hasta los vertices del 4ABC, se ubican sobre el circuncrculo del
4A0 B 0 C 0 .
d) Si N es el circuncentro del 4A0 B 0 C 0 , demuestre que N , O, G forman la recta de Euler
del 4A0 B 0 C 0 y utilice sus propiedades para probar que N es el punto medio de HO.

68. El area del 4ABC, denotada por [ABC], cumple:

base altura
[ABC] =
2
ab senC bc senA ca senB abc
[ABC] = = = =
2 2 2 4R
[ABC] = sr
p
[ABC] = s(s a)(s b)(s c). (Formula de Heron).
32
Tambien conocida como Circunferencia de Feuerbach.

70
69. El circunradio, el inradio y los exradios de un triangulo cumplen:

4R = ra + rb + rc r
[ABC] = ra (s a) = rb (s b) = rc (s c)
r
(s a)(s b)(s c)
r =
r s
s(s b)(s c)
ra =
sa
r
s(s a)(s c)
rb =
sb
s
s(s a)(s b)
rc =
(s c)

70. Dado el 4ABC, sea I el incentro e Ia el excentro respecto a A.

a) Demuestre que BICIa es un cuadrilatero cclico.


b) Si M es la interseccion de IIa con el circuncrculo del triangulo (M 6= A), demuestre
que dicho punto es el circuncentro de BICIa .
c) Sea M 0 el punto diametralmente opuesto a M en el circuncrculo, y sea P la proyeccion
de I sobre AB. Demuestre que 4M 0 CM ' 4AIP .
d) Sea O el circuncentro del triangulo Calcule la potencia de punto de I con respecto
al circuncrculo, y utilizando los resultados anteriores deduzca la Formula de Euler :
OI 2 = R2 2Rr.
e) A partir de la formula de Euler demuestre que R 2r.

71
8. Solucion a Problemas Selectos.
Teoremas Fundamentales del Triangulo.
1. En la figura 94, ABDE es un cuadrado y BCD es un triangulo isosceles con BD = DC.
Si ABC = 160, determinar la medida de AEC.

Figura 94

Solucion: DBC = DCB = 160 ABD = 70, de donde se obtiene que BDC = 40
y EDC = EDB + BDC = 130. Como 4EDC es isosceles, entonces DEC =
DCE = 25. Por lo tanto AEC = 90 DEC = 65.

2. Hallar la suma de los angulos +  + + en la figura 95.

Figura 95

Solucion: CAB = , EDC = por ser opuestos por el vertice. Como el AF D


externo en el 4BDF , se tiene AF D = + . Sumando los angulos internos del 4AEF
se tiene + + +  = 180.

3. (XV Competencia de Clubes Cabri Primera Ronda) En la figura 96, ABCD es un


rectangulo tal que AB = 2BC. M es el punto medio de AB y los triangulos AM E
y M BF son equilateros. Si P es la interseccion de las rectas DE y CF , encuentre los
angulos del 4CDP .

72
Figura 96

Solucion: Note que AD = AE = F B = BC por lo que 4DAE y 4BCF son ambos


isosceles. Luego DAE = CBF = 90+60 = 150 lo que implica que P DC = P CD =
90 15 = 75 y luego CP D = 30.

4. Sea ABC un triangulo rectangulo con CAB = 90 (Ver figura 97). D es un punto sobre
la prolongacion de BC tal que BD = BA. E es un punto en el mismo semiplano que A
respecto de BC, tal que CE BC y ademas CE = CA. Mostrar que A, D y E estan
alineados.

Figura 97

Solucion: Sea CBA = 2; el 4ABD es isosceles y BAD + BDA = 2, por lo


que BAD = BDA = . Como CAB = 90 entonces ACB + ABC = 90 y como
CE es perpendicular a BC entonces ECA + ACB = 90; por lo tanto, ABC =
ECA = 2. Con esto, como 4ECA es isosceles, CEA = CAE = 90 . Luego,
EAC + CAB + BAD = 180 y as E, A y D estan alineados.

5. Dado un cuadrado ABCD, se construyen los triangulos equilateros ABP (exteriormente)


y ADQ (interiormente). Probar que C, P y Q estan alineados. Figura 98.

73
Solucion: Observe que P AQ = BAD = 90 y P A = BA = DA = DQ, por lo
que 4P AQ es triangulo rectangulo isosceles, y por tanto, P QA = 45. Por otra parte,
QDC = 90 ADQ = 30 y QD = AD = CD, es decir, el 4CDQ es isosceles
con el angulo comprendido entre lados iguales de 30, por lo que DQC = 75. As,
P QA + AQD + DQC = 180 y por lo tanto, C, P y Q estan alineados.

Figura 98

6. En la figura 99, AB = BC = CD = DE = EF = F G = GA. Calcule la medida del


DAE. Referenciasfig20

Figura 99

Solucion: Sea DAE = . Como los triangulos ABC y AGF son isosceles, ACB =
AF G = . Calculando los angulos externos de 4ABC y 4AF G se tiene F BC =
CGF = 2. Como 4GF E y 4BCD son isosceles, GEF = BDC = 2. Calculando
angulos externos de 4ADC y 4AEF se obtiene ECD = DF E = 3. Como 4CDE
y 4F ED son isosceles, CED = F DE = 3. Entonces, la suma de los angulos internos
del 4AED da + 3 + 3 = 180, de donde = 180
7
.

7. (XXVIII Olimpiada Brasilena de Matematica) En la figura 100, AB = AC, AM = AN


y CAM = 30, encuentre el valor del BM N .
Solucion: Como 4ABC y 4AM N son isosceles, sean ABC = ACB = y AM N =
AN M . Por la formula del angulo externo se tiene

74
Figura 100

ACM + M AC = AM B = AM N + BM N
+ 30 = AN M + BM N
= (N BM + BM N ) + BM N
= + 2BM N
Esto implica que BM N = 15.

8. (Etapa semifinal Estatal de XXII Olimpiada Mexicana de Matematicas) En la figura


101 se muestra un hexagono regular ABCDEF de lado 1. Los arcos del crculo que estan
dibujados tienen centro en cada vertice del hexagono y radio igual a la distancia al vertice
opuesto. P , Q, R, S, T y U son los puntos de corte de estos arcos. Cuanto mide cada
lado del hexagono P QRST U ?

Figura 101

Solucion: El hexagono P QRST U es regular y con el mismo centro que ABCDEF . Sea
O el centro de ambos (Vease Figura 64). El lado buscado es igual a OP . Tenemos que
CF = F P = P C = 2 por ser radios de los arcos dibujados; entonces CF P esequilatero
de lado 2 y OP es una altura de este triangulo que, por Pitagoras, es igual a 3.

75
Congruencia de Triangulos
1. En la figura 102, ABC es un triangulo equilatero y CDEF es un cuadrado. Se construye
un punto G tal que CF = CG y ademas CF G = 15. Probar que AGC = BDC.

Figura 102

Solucion: BCD = 180 ACB DCF = 30. Como 4GCF es isosceles, CGF =
CF G = 15 y ACG = CGF + CF G = 30. Por criterio LAL, 4BCD 4ACG,
por lo tanto BDC = AGC.

2. (Cuaderno de Olimpiadas Mexicanas - Geometra) En la figura 103, ABCD un cuadrado


y EF GH. Demuestre que que EF = GH.

Figura 103

Solucion: Se construyen EK y GM con K sobre CD y M sobre AD tales que EK k AD


y GM k CD. Luego se demuestra que 4EF K 4GHM , con EF = GH.

3. (Examen final de XVI Olimpiada mexicana de Matematica) Los angulos de un triangulo


ABC estan en progresion aritmetica (B A = C B = ), D, E, y F son los
puntos medios de los lados BC, CA y AB, respectivamente. Llamamos H al pie de la
altura trazada desde C (que cae entre B y F ) y G a la interseccion entre DH y EF .
Cuanto vale F GH?

Solucion: Note que A + B + C = 3A + 3 = 180, lo cual implica que A + =

76
60 = B. Entonces 4BCH es un triangulo 30, 60, 90, y dado que D es punto medio de
BC, el 4BDH es equilatero. Luego, como BC k EG, F GH = BDH = 60. Ver figura
104.

Figura 104

4. Sea ABCD un cuadrado. Se construyen triangulos equilateros ADP y ABQ como se


muestra en la figura 105. Sea M la interseccion de CQ con AD y N la interseccion de
CP con AB. Demuestre que CM N es un triangulo equilatero.

Figura 105

Solucion: Note que P D = AD, porque 4AP D es equilatero, y AD = CD porque


ABCD es cuadrado, por lo que P D = CD, es decir, el 4CDP es isosceles, con CDP =
CDA P DA = 30, entonces DP C = DCP = 75, y BCN = BCD DCP =
15. Analogamente, 4BCQ es isosceles con angulos 30, 75, 75, por lo que M CN =
BCQ BCN = 60. Finalmente, como la figura es simetrica con respecto a AC,
CM = CN , entonces, el triangulo CM N es equilatero porque tiene dos lados iguales y
un angulo interno igual a 60.

5. 4ABC es un triangulo isosceles con ABC = ACB = 80. D es un punto en AC tal


que ABD = 10. Demuestre que AD = BC. Figura 106.

77
Solucion: Se traza un punto D0 sobre AC tal que AD0 = BC. Se construye exteriormente
el triangulo equilatero AEB. Luego, AE = AB, D0 A = CB y EAD0 = ABC lo
cual implica que 4EAD0 4ABC, de donde se deduce que el 4D0 EB es isosceles
y BED0 = BEA D0 EA = 40. Se sigue que EBD0 = 70 y como D0 BA =
EBD0 ABE = 10, resulta que D0 = D y por lo tanto BC = AD0 = AD.

Figura 106

78
Cuadrilateros.
1. Sea ABCD un paralelogramo. Se construyen triangulos equilateros exteriores 4CDP y
4ADQ, como se muestra en la figura 107. Demuestre que el 4BP Q es equilatero.

Figura 107

Solucion: Observe que al hacer una rotacion de centro P y angulo 60, el triangulo
P BC se transforma en el triangulo P QD (observe los segmentos P C y CB tras esta
transformacion), mientras que al hacer una rotacion de centro Q y angulo 60, el triangulo
P QD se transforma en triangulo BQA. Como la rotacion mantiene las distancias, P B =
P Q = BQ, por lo que el trangulo BP Q es equilatero.33

2. (II Olimpiada Matematica del Cono Sur) En la figura 108 ABCD y AECF son parale-
logramos. Demuestre que BEDF es paralelogramo.

Figura 108
33
Una demostracion mas rigurosa se basa en el calculo de los angulos P CB = P DQ = BAQ = 120 +
ABC y en la utilizacion del criterio LAL para justificar 4P CB 4P DQ 4BAQ.

79
Solucion 1: Sea M el punto medio de AC. Las diagonales AC y BD se bisecan en M ,
mientras que las diagonales AC y EF tambien se bisecan en M , entonces BD y EF se
bisecan en M por lo que BEDF es un paralelogramo.
Solucion 2: Como AD k CB y AE k CF entonces DAE = BCF . Entonces, por
propiedades de paralelogramos BAE = BAD EAD = BCD BCF = F CD;
ademas, AB = CD y AE = CF . Por criterio LAL, 4BAE 4DCF , y entonces BE =
DF . Analogamente se demuestre que 4ABF 4CDE, lo cual implica BF = DE.
Como BEDF es un cuadrilatero con lados opuestos iguales, es un paralelogramo.

3. ABCD es un cuadrilatero convexo y O es un punto en su interior. Sean P , Q, R, S,


los puntos medios de los lados AB, BC, CD, DA, respectivamente. Por P se traza una
paralela a OR, por Q se traza una paralela a OS, por R se traza una paralela a OP , y
por S se traza una paralela a OQ. Demuestre que estas cuatro rectas concurren.

Solucion: Al tomar las rectas OP y OR y sus paralelas se forma el paralelogramo P ORM ,


y al tomar las rectas OQ y OS y sus paralelas se forma el paralelogramo OQN S. Por el
teorema de Varignon, sabemos que P QRS es un paralelogramo, y llamaremos T al punto
de corte de sus diagonales. Observe que el punto de corte de las diagonales de P ORM es
el punto medio de P R, i.e., T ; analogamente, el punto de corte de las digonales de OQN S
es el punto medio de SQ, i.e., T nuevamente. As, M es la reflexion de O con respecto a
T , y de igual forma queda definido N , por lo que M = N y las cuatro rectas concurren.

4. (Hector Alberti) Sea ABCD un cuadrado. Se construyen los triangulos equilateros BDA0 ,
ACB 0 , BDC 0 y ACD0 (Vease figura 109). Demuestre que el A0 B 0 C 0 D0 es tambien un
cuadrado.

Figura 109

Solucion: Como A0 B = A0 D, AB = AD, CB = CD, C 0 B = C 0 D, los puntos A0 , A,


C, C 0 pertenecen a la mediatriz de BD, y por tanto, estan alineados. Analogamente, B 0 ,
B, D, D0 estan alineados; por lo tanto A0 C 0 B 0 D0 . Por otra parte, si O es el centro
de ABCD, como los triangulos equilateros construidos son todos iguales (tienen lados
iguales a la diagonal de ABCD) de altura h, OA0 = OB 0 = OC 0 = OD0 = h. Entonces,
A0 B 0 C 0 D0 es un cuadrilatero con diagones que se bisecan en O (es paralelogramo), son
iguales A0 C 0 = B 0 D0 = 2h (es rectangulo) y son perpendiculares (es rombo), lo cual
implica que es cuadrado.

80
5. Un trapecio isosceles tiene diagonales perpendiculares y su area es 2010, determine su
altura.

Solucion: Considere la figura 110. Sea ABCD el trapecio del problema (AB = CD),
como es trapecio isosceles, es simetrico con respecto a la mediatriz de las bases, en parti-
cular, AC = BD. Sean P y Q los pies de las perpendiculares a AD trazadas desde B y
C, respectivamente. Por LAL, 4ABD 4DCA lo cual implica CAD = BDA = 45
(debido a que AC BD) luego en el triangulo rectangulo 4ACQ, ACQ = 45 por lo
que AQ = CQ y es facil ver que BC = P Q. Luego

(CQ)(BC + AD)
2010 = (ABCD) =
2
(CQ)(P Q + AP + P Q + QD)
=
2
= (CQ)(AP + P Q)
= CQ2

Luego CQ = 2010.

Figura 110

6. (IX Competencia de Clubes Cabri, Segunda Ronda) Sea ABCDEF un hexagono regular
cuyo centro es O. Se construyen los cuadrados F SOP y ORCQ. Demuestre que AP QB
y SEDR son rectangulos. Figura 111.

Solucion: Por construccion P F = P O = SF = SO, y por propiedades de hexagono


regular34 AF = AO = EF = EO, entonces P , S A, E, pertenecen a la mediatriz de F O
y por tanto, estan alineados sobre una recta perpendicular a F O. Analogamente, Q, R,
B, D estan alineados sobre una recta perpendicular a CO; ademas, es facil demostrar
que AP = BQ = DR = ES. Observe ademas que AB k CF k DE, lo cual implica
(AB k CF ) (AP k BQ), es decir, AP QB es rectangulo, y analogamente para SEDR.

34
Los triangulos OAB, OBC, OCD, ODE, OEF , OF A son equilateros.

81
Figura 111

7. Sobre los lados del 4ABC se trazan exteriormente los cuadrados ABP Q, CARS y
BCT U . Luego se trazan los paralelogramos AQA0 R, CSC 0 T y BU B 0 P , como en la figura
112.

a) Sean A00 , B 00 , C 00 los centros de los cuadrados BCT U , CARS, ABP Q, respectiva-
mente. Demuestre que estos centros estan sobre los lados del 4A0 B 0 C 0 .
b) Demuestre que AA00 , BB 00 , CC 00 concurren.

Solucion:

a) Observe que al hacer una rotacion de centro A00 y angulo igual a 90, el 4A00 U B 0 se
transforma en el 4A00 BA, y a la vez este ultimo se transforma en el 4A00 CC 0 (esto es
porque A00 U 7 A00 B 7 A00 C y U B 0 7 BA 7 CC 0 ); esto significa que A00 B 0 A00 A
y A00 A A00 C 0 , por lo que B 0 , A00 , C 0 estan alineados, es decir, A00 pertenece a B 0 C 0 .
Analogamente se prueban los otros casos.
b) De lo anterior, observe que AA00 es mediatriz de B 0 C 0 , por lo que AA00 , BB 00 , CC 00
concurren en el circuncentro del 4A0 B 0 C 0 .

8. Se dibujan cuadrados exteriores a los lados de un paralelogramo (Vea figura 113), de-
muestre que:

a) El cuadrilatero determinado por los centros de esos cuadrados es un cuadrado.


b) Las diagonales de ese cuadrado son concurrentes con las del paralelogramo.

Solucion:

82
Figura 112

a) Observe que al hacer una rotacion de centro O2 y angulo igual a 90, el 4O2 BO1 se
transforma en el 4O2 CO3 (observe que los segmentos O2 B y BO1 se transforman
en O2 C y CO3 , respectivamente), por lo que O2 O1 = O2 O3 y O2 O1 O2 O3 . Re-
pitiendo este razonamiento, O1 O2 = O2 O3 = O3 O4 = O4 O1 y estos segmentos son
perpendiculares si son consecutivos, por lo que O1 O2 O3 O4 es un cuadrado.
b) Basta demostrar que AC y O1 O3 se bisecan,35 y esto es equivalente a demostrar
que AO1 CO3 es un paralelogramo. Esto es cierto porque AO1 = CO3 y AO1 k CO3
(ambos segmentos son perpendiculares a O1 B)

Figura 113

35
Porque as los puntos de corte de las diagonales de ABCD y O1 O2 O3 O4 coincidiran.

83
9. Dado un 4ABC, se construyen exteriormente los triangulos rectangulo isosceles 4ACP
y 4BCQ, con AC y BC como hipotenusas. Si M es el punto medio de AB, demuestre
que el 4M P Q tambien es un triangulo rectangulo isosceles.

Solucion: Construya los cuadrados exteriores ACDE y BCF G, como muestra la figura
114. Observe que P y Q son los puntos medios de AD y BF , respectivamente. Al rotar
el 4BCD con centro C y angulo de 90, se genera el 4F CA, entonces dichos triangulos
son congruentes y en por tanto BD = AF y BD AF . Por otra parte, observe que M P
es base media del 4BAD, por lo que 2M P = BD y M P k BD; analogamente, M Q es
base media del 4ABF , por lo que 2M Q = AF y M Q k AF . Por lo tanto M P = M Q y
M P M Q.

Figura 114

84
Angulos en Circunferencia.
1. Dada la figura 115, demuestre que AB k A0 B 0 .

Figura 115

Solucion: Observe que los cuadrilateros ABQP y A0 B 0 QP son cclicos, por lo que
P AB = P QB 0 = 180P A0 B 0 , por lo tanto AB k A0 B 0 .
2. Dos circunferencias de centros O1 y O2 son tangentes (interna o externamente) en un
punto P ; por este punto se traza una recta que corta nuevamente a la circunferencias en
A y B, respectivamente. Demuestre que AO1 k BO2 .

Solucion: En la figura 116 se ha considerado que las circunferencias son tangentes exte-
riormente, sin embargo, el otro caso es analogo. Se sabe que O1 , P , O2 estan alineados, y
que 4AP O1 y BP O2 son triangulos isosceles (dos de sus lados son radios de una circun-
ferencia), entonces O1 AP = AP O1 = BP O2 = O2 BP , por lo que AO1 k BO2 .

Figura 116

3. Dadas dos circunferencias una fuera de la otra como en la figura 117, demuestre que las
tangentes comunes externas forman segmentos iguales; analogamente, las tangentes co-
munes internas forman segmentos iguales.

Solucion:36 Sea P la interseccion de las tangentes comunes externas AA0 y BB 0 . Entonces


36
Suponemos que las circunferencias tienen radios distintos; cuando los radios son iguales, el problema se
justifica por la simetra de la figura.

85
AA0 = P A0 P A = P B 0 P B = BB 0 . Analogamente se resuelve el caso de las tangentes
comunes internas.

Figura 117

4. Teorema de Pithot. Demuestre que en todo cuadrilatero inscribible, la suma de lados


opuestos es igual.37

Solucion: Considere la figura 118, ABCD es el cuadrilatero inscribible, con P , Q, R, S,


los puntos de tangencia sobre AB, BC, CD, DA, respectivamente. Entonces

AB + CD = AP + P B + CR + RD
= AS + BQ + CQ + DS
= BC + DA

Figura 118: Teorema de Pithot.

5. Teorema de Steiner. En todo cuadrilatero exinscrito a una circunferencia, la diferencia


de las longitudes de lados opuestos es igual.

Solucion: El cuadrilatero puede quedar en posiciones como las de la figura ??; en ambos
casos, la demostracion es muy similar, y analoga a la de Pithot. Para la figura de la
37
El recproco de este teorema y del siguiente son tambien es ciertos.

86
izquierda se tiene que

AB CD = (AP BP ) (CR RD)


= (AS BQ) (CQ DS)
= AD BC

Figura 119: Teorema de Steiner.

6. Teorema de Miquel: Dado un 4ABC, sean X, Y , Z puntos sobre AB, BC, CA,
respectivamente . Demuestre que los circuncrculos de 4AXZ, 4BY X, 4CZY tienen
un punto en comun M .
Solucion: Sea M el otro punto de corte de los circuncrculos de 4AXZ y 4BY X. Como
los cuadrilateros AXM Z y BY M X son cclicos, se tiene

Y M Z = 360 XM Z Y M X
= 360 (180 A) (180 B)
= 180 C

Entonces, CY M Z es cuadrilatero cclico, por lo que M esta sobre el circuncrculo del


4CZY .

Figura 120: Teorema de Miquel.

87
7. Sea ABC un triangulo, y sean L y N las intersecciones de la bisectriz del angulo A con
el lado BC y el circuncrculo de ABC respectivamente (Ver figura 121). Construimos la
interseccion M del circuncrculo de ABL con el segmento AC. Prueba que los triangulos
BM N y BM C tienen la misma area.

Solucion: Observe que ABN C y ABLM son cuadrilateros cclicos, por lo que N CB =
N AB = LAM = LBM , por lo que CN k BM . Entonces, las distancias de N y C a
la recta BM son iguales, y por tanto, el area del 4BM N es igual al area del 4BM C.

Figura 121

8. Sea AB el diametro de una semicircunferencia. Se colocan los puntos M y K sobre la


semicircunferencia y sobre AB, respectivamente.38 Sea P el centro de la circunferencia
que pasa por A, K y M ; sea Q el centro de la circunferencia que pasa por B, K y M .
Demuestre que M P KQ es concclico.
Solucion: Como AB es diametro, AM B = 90, entonces M AB + M BA = 90. As

M P K + M QK = 2M AK + 2M BK
2 = 2 (M AB + M BA)
= 180

Por lo tanto, M P KQ es concclico.

38
M y K son distintos de A y B.

88
9. Las circunferencias 1 y 2 se cortan en los puntos A y B. Por el punto A se traza una recta
que corta nuevamente a las circunferencias 1 y 2 en los puntos C y D, respectivamente.
Por los puntos C y D se trazan tangentes a las circunferencias, las cuales se cortan en el
punto M . Demuestra que M CBD es cclico. Figura 122.
Solucion: Es suficiente probar que M CBD es un cuadrilatero con un par de angulos
opuestos suplementarios. Por angulos seminscritos y suma de angulos internos de un
triangulo, se tiene

CM D + CBD = CM D + CBA + DBA


= CM D + M CA + M DA
= 180

Figura 122

10. El 4ABC cumple que A = 90 y AB = AC. Se toma un punto E del segmento AB, se
construye interiormente un triangulo equilatero AEF . EF corta BC en I, y se construye
exteriormente un triangulo equilatero BIJ. Encuentre EJB.

Solucion: Como el BJI = 60 = AEI,


el cuadrilatero BEIJ es cclico, por lo que el
EJB = EIB = AEI EBI = 15.

89
11. En la figura 123, se sabe que AO1 B AO2 B = 70 y ademas la tangente EB forma
el triangulo isosceles ABE, con AB = AE. Encuentre EBC.

Figura 123

Solucion: Sea AO2 B = 2, entonces ACB = y por hipoteis AO1 B = 2 + 70. Por
angulo seminscrito, ABE = + 35, y como el 4ABE es isosceles, AEB = + 35.
Finalmente, por la formula del angulo externo aplicada al 4BCE, EBC = AEB
ECB = 35

12. Dos circunferencias 1 y 2 se cortan en A y B. Una recta por A corta a 1 y 2 en C y


D, respectivamente, y la paralela a CD por B corta 1 y 2 en E y F , respectivamente.
(Ver figura 124). Demuestre que 4CDB 4EAF .
Solucion: Sean G = AE BC y H = AF BD. Como AC k BE y ACEB es ccli-
co,39 CAG = GEB = ACG = GBE = ; analogamente, DAH = HF B =
ADH = F BH = . Observe que 4GAC y 4GBE son triangulos isosceles y por tan-
to AE = AG+GE = CG+GB = CB; de forma similar se obtiene AF = DB. Finalmente,
EAF = 180 = CBD, por lo que, por el criterio LAL, 4CBD 4EAF .

13. La Recta de Simson-Wallace. Sean X, Y y Z los pies de las alturas trazadas desde un
punto P en el circuncrculo del 4ABC hacia AB, BC y CA, respectivamente. Demuestre
que X, Y y Z estan alineados.40

Solucion: Como BP Y X es cclico, Y XP = Y BP = . Como ABP C es cclico,


CBP = CAP = . Como AXP Z es cclico, ZAP = ZXP = . Por lo tanto,
dado que Y XP = ZXP , los puntos X, Y y Z estan alineados.
39
ACEB es un trapecio isosceles.
40
El recproco tambien es cierto, si X, Y y Z estan alineados, entonces P debe estar sobre el circuncrculo
del 4ABC; en cualquier otro caso, el 4XY Z se llama el triangulo pedal con respecto al punto P .

90
Figura 124

Figura 125: Recta de Simson-Wallace

14. Sea P un punto exterior al cuadrado ABCD tal que AP C = 90 , Q es la interseccion


de AB y P C, y R el pie de la perpendicular por Q a CA. Demuestre que P , R y D estan
alineados.

Solucion: Como AP C + ADC = 180, el cuadrilatero P ADC es cclico, entonces


AP D = ACD = 45. Analogamente, como AP Q + ARQ = 180, el cuadrilatero
P ARQ es cclico, entonces AP R = AQR = 90QAR = 45. Por lo tanto, como
AP D = AP R, los puntos P , R, D estan alineados. Figura 126.

15. (OIM 2002, P-4) En un triangulo escaleno ABC se traza la bisectriz interior BD, con D
sobre AC. Sean E y F puntos sobre la recta BD tales que (AE k CF ) BD, y sea M el
punto sobre el lado BC tal que DM BC. Demuestre que EM D = DM F . Figura
127.

91
Figura 126

Solucion: Como DM M C y DF F C, DF CM es cclico, por lo tanto DM F =


DCF = , y como AE k F C, entonces EAD = DCF = . Sea G la interseccion
de AE con BC. Como AG BE, BE es altura y bisectriz del 4ABG, por lo que este
triangulo es isosceles y ademas BE es mediatriz de AG; entonces EGD = EAD = .
Y finalmente, podemos ver que DEM G es cclico, pues DEG = DM G = 90 , as que
EM D = EGD = . De aqu, el resultado es inmediato.

Figura 127

16. (OMCC 2003, P-2) Sea S una circunferencia y AB un diametro de ella. Sea t la recta
tangente a S en B y considere dos puntos C y D en t tales que B este entre C y D. Sean
E y F las intersecciones de S con AC y AD y sean G y H las intersecciones de S con
CF y DE. Demuestre que AH = AG.

Solucion: Como AEBF es cclico (Ver figura 128), AEF = ABF . Luego, como AB
CD y BF AD, se cumple tambien ABF = F DB, por lo que AEF = F DC, es
decir, el cuadrilatero CDF E es cclico. Utilizando este resultado y el hecho que EGHF
tambien es cclico, se tiene EDC = EF G = EHG, por lo que CD k GH. Esto

92
implica que AB GH, y como AB pasa por el centro de la circunferencia, debe ser
mediatriz de GH, por lo tanto AG = AH.

Figura 128

17. (The 59th Romanian Mathematical Olympiad District Round) Considere un cuadrado
ABCD y un punto E sobre el lado AB. La diagonal AC corta al segmento DE en el pun-
to P . La perpendicular por P a DE corta al lado BC en F . Probar que EF = AE + CF .

Solucion: Se construye E 0 sobre BC de tal manera que CE 0 = AE (como se mues-


tra en la figura 129) y que C quede entre F y E 0 , as por LAL se tiene que los triangulos
rectangulos 4DAE 4DCE 0 por lo tanto ADE = CDE 0 luego EDE 0 = 90. Por
otra parte, el cuadrilatero DCF P es cclico, por lo que P DF = P CF = 45 entonces
F DE 0 = EDE 0 EDF = 45. Ahora por LAL los triangulos 4DEF 4DE 0 F ,
por lo que EF = E 0 F = E 0 C + CF = AE + CF .

Figura 129

93
18. Teorema de Arqumedes: En la figura 130, la region delimitada por tres semicircunfe-
rencias mutuamente tangentes, es conocida como cuchilla de zapatero o arbelos. Demostrar
que las circunferencias sombreadas son congruentes.

Figura 130: Teorema de Arqumedes.

Solucion: Sean AB, AC, BC los diametros de las semicircunferencias que forman el arbe-
los, de radios r, r1 , r2 y centros O, O1 , O2 , respectivamente. De momento nos concentramos
en el lado izquiero de la figura Referenciasfigura58; sea C1 el centro de la circunferencia de
la izquierda y R1 su radio; D, E y F son los puntos de tangencia de esta circunferencia con
dos semicircunferencias del arbelos y con la recta perpendicular a los diametros por C; fi-
nalmente, G es la proyeccion de C1 sobre AB. En primer lugar, OO1 = OAO1 A = rr1 .
Por otra parte, observe que O, C1 , D estan alineados y O1 , C1 , E tambien estan alineados,
entonces OC1 = OD C1 D = r R1 y O1 C1 = O1 E + EC1 = r1 + R1 . Ademas, como
CF C1 G es un rectangulo, GC = F C1 = R1 , entonces O1 G = O1 C GC = r1 R1 y
OG = O1 G O1 O = (r1 R1 ) (r r1 ) = 2r1 (r + R1 ). Ahora, aplicando el teorema
de Pitagoras a 4GO1 C1 y 4GOC1 se tiene
O1 C12 O1 G2 C1 G2 = OC12 OG2 C1 G2
(r1 + R1 )2 (r1 R1 )2 = (r R1 )2 (2r1 (r + R1 ))2
4r1 R1 = 4rR1 4r12 + 4r1 r + 4r1 R1
r1 (r r1 )
R1 =
r

Figura 131

Analogamente, si r2 y R2 son los radios del semicrculo y del crculo de la derecha, res-
pectivamente, entonces
r2 (r r2 )
R2 =
r

94
Pero 2r = AB = AC + BC = 2r1 + 2r2 , entonces r2 = r r1 , y sustituyendo en la
ecuacion anterior se tiene
(r r1 ) (r (r r1 )) (r r1 )r1
R2 = = = R1
r r

95
Teorema de Thales y Semejanza.
1. (IV OMCC, P-4) Sea ABC un triangulo, D el punto medio de BC, E un punto sobre
el segmento AC tal que BE = 2AD y F el punto de interseccion de AD con BE. Si
CAD = 60, encuentre la medida de los angulos del 4F EA. Figura 132.

Solucion: Se traza por D una paralela a BE y sea G el punto por el que esta paralela
corta al lado AC. Como DG es base media del 4BCE se tiene que DG = BE 2
= AD;
entonces 4ADG es isosceles y tiene un agulo de 60, por lo que debe ser equilatero.
Finalmente 4AEF tambien es equilatero, por tanto sus angulos son iguales a 60.

Figura 132

2. Sea ABCD es un trapecio con AD k BC. M y N son los puntos medios de CD y BC,
respectivamente, y P el punto comun de las rectas AM y DN . Si PAP
M
= 14 , demuestre que
ABCD es paralelogramo.

Solucion: Sea Q el punto medio de DN , entonces QM k BC k DA. Como M Q es


base media del 4CDN , M Q = CN 2
= CB4
. Por otra parte, como 4P M Q ' 4P AD,
MQ PM 1 AD
AD
= AP = 4 , entonces M Q = 4 . Finalmente, como BC k DA y BC = DA, ABCD
es paralelogramo.

3. En la figura 133, BC = CD = DE = EA = x y AEB = 90. Demuestre que ABC +


ACD + ADE = 90.

Figura 133

Solucion 1: Por Pitagoras, AD = 2x. Observe que DA2 = 2x2 = DB DC, por lo que
4ABD ' 4CAD; entonces ABD = CAD y por tanto ABC + ACD + ADE =

96
CAD + ACD + ADE = 2ADE = 90.

Solucion 2: Considere la siguiente cuadrcula (Figura 134). Observe que al hacer una
rotacion de centro A y angulo igual a 90, el segmento AC se transforma en AF , por lo
que el 4ACF es triangulo rectangulo isosceles, y ADE = ACF . Se cumple ABC =
CF D, porque se forman con la diagonal de tres cuadrados; analogamente, ACD =
AF D, porque se forman con la diagonal de dos cuadrados. Sumando los angulos internos
del 4ACF se obtiene el resultado buscado.

Figura 134

4. (Asiatico Pacfica) Sea ABC un triangulo y D el pie de la altura con respecto a A. Sean E
y F puntos en una recta que pasa por D (distintos de D) tales que AE CE y AF BF .
Sean M y N los puntos medios de BC y EF , respectivamente. Demuestre que AN N M .

Solucion: En la figura 135, AE CE y AD DC entonces, ADEC es cclico, as que


DEA = DCA. Del mismo modo, como AF BF y AD DB, AF BD es cclico
y entonces AF D = ABD. Esto implica que 4ABC ' 4AF E y a partir de esta
semejanza, 4ABM ' 4AF N . Luego, AM B = AN F , por lo que el cuadrilatero
AN DM es cclico, y por lo tanto AN M = ADM = 90.

97
Figura 135

Puntos y rectas notables.


1. De acuerdo con los datos de la grafica 136, calcular el valor de AB.

Figura 136

AC CD
Solucion 1: Por el teorema de la bisectrz AB = DB , de donde AC = 45 x, luego, aplicando
2
el teorema de pitagoras al 4ABC, se tiene que x2 + 182 = 54 x , que despues de resolver
se tiene que x = 24.
Solucion 2: Dado que D es un punto de la bisectrz del BAC, entonces D equidista
de los lados de dicho angulo, sea pues H AC talque DHAC y DH = DB = 8
entonces, aplicando el teorema de pitagoras en el 4CDH se deduce que HC = 6, por lo
que AC = x + 6, y por el teorema de pitagoras en el 4ABC, x2 + 182 = (x + 6)2 , por lo
tanto, x = 24.
2. Sea ABCD un paralelogramo. Q es el punto medio de AD, F el pie de la perpendicular
por B sobre QC. Probar que AF = AB.
Solucion: Sea E el punto medio de BC y G la interseccion de AE con BF . Como
AE k CQ, se tiene que AG BF . Pero tambien, como AE k CQ, entonces EG k CF
por lo que en el 4BCF , EG es base media. Entonces BG = GF de donde se sigue que
4ABF es isosceles porque BG es altura y mediana.

98
3. Sea ABCD un cuadrilatero tal que AB = CD. Las mediatrices de AC y BD se cortan
en P . Probar que P AC = P CA = P BD = P DB. Figura 137.
Solucion: Como P esta sobre las mediatrices de AC y BD, P A = P C y P B = P D,
y por hipotesis, AB = CD, entonces por criterio LLL, 4ABP 4CDP . De aqu,
AP B = CP D, entonces BP D = AP D + AP B = AP D + CDP = AP C;
por lo tanto, 4BP D ' 4CP A, dada la igualdad anterior y el hecho que son triangulos
isosceles. De esta semejanza se obtiene P AC = P CA = P BD = P DB.

Figura 137

4. ABC es un triangulo y P un punto en su interior. Sean A0 , B 0 y C 0 las reflexiones de


P sobre BC, CA y AB, respectivamente. D, E y F son los pies de las perpendiculares
respectivos desde A, B y C hacia B 0 C 0 , C 0 A0 y A0 B 0 . Probar que AD, BE y CF son
concurrentes. Figura 138.

Solucion: Por propiedades de reflexion axial AC 0 = AP = AB 0 , por lo que el 4AB 0 C 0 es


isosceles, y entonces AD es mediatriz de B 0 C 0 . Analogamente, BE es mediatriz de C 0 A0 ,
mientras que CF es mediatriz de A0 B 0 . Por lo tanto, las rectas AD, BE, CF concurren
en el circuncentro del 4A0 B 0 C 0 .

5. (Arnoldo Aguilar) En la figura 139, ABGH, BCF G y CDEF son cuadrados. Si I es el


centro de ABGH y J = DH BG, demuestre que I, J y F estan alineados.
Solucion: Como G es punto medio de HF , BG es una mediana del 4BF H. Ademas,
BDF H es un paralelogramo, luego sus diagonales BF y DH se cortan en su punto medio,
digamos K. Se sigue que HK es tambien una mediana del 4BF H, y en consecuencia el
punto de corte de J = KH BG es el centroide del 4BF H. Pero I es el punto medio de
BH, as que F I es la tercera mediana del 4BF H, por lo tanto J esta sobre el segmento
F I.

99
Figura 138

Figura 139

6. (Arnoldo Aguilar) Sea ABC un triangulo equilatero. M y N son los puntos medios de AB
y BC, respectivamente. Exteriormente al 4ABC se construye un triangulo rectangulo
isosceles 4AP C, con AP C = 90 . Si I es la interseccion de AN y M P , demuestre que
CI es la bisectriz de ACM .

Solucion: Observe que AN es bisectriz del BAC.


Como AP C = BM C = 90, el cuadrilatero
AP CM es cclico, por lo que P M C = P AC =
P CA = P M A = 45, entonces M P es bisectriz
del AM C. De aqu se concluye que I = M P AN
es el incentro del 4ACM , por lo que CI es bisectriz
del ACM .

7. En la figura 140, el 4ABC es tal que A = 90 y B = 60. Cual es el radio de la


circunferencia?

Solucion: Por relaciones de triangulos notables, BC = 2 y CA = 3. Sean P y Q
las proyecciones de O sobre AB y AC respectivamente; por construccion, AP OQ es un
rectangulo, pero como OP = r = OQ, es tambien cuadrado, por lo que AP = r. Observe

100
Figura 140

que la circunferencia es el excrculo del 4ABC, por lo que AP = s y entonces

r = s
1+2+ 3
=
2
3+ 3
=
2

8. Dado el paralelogramo ABCD, sea M el punto medio de AB, y N la interseccion de CD


con la bisectriz interna del ABC. Demuestre que M C BN si y solo si AN es bisectriz
del DAB.

Solucion:
() Si suponemos que M C BN entonces BN es mediatriz de M C, y como BM k CN
entonces CBN = M BN = CN B = M N B, esto implica que BC k M N , y por
tanto N es punto medio de CD; as, AM N D es un rombo y AN es bisectriz del DAM .
() Si suponemos que AN es bisectriz del DAB, es propiedad conocida que AN BN ,
por lo que M es el circuncentro del 4ABN y por la relacion entre angulo central y angulo
inscrito se tiene AM N = 2ABN = ABC, por lo tanto M N k BC y BCN M es un
rombo, de donde se obtiene M C BN .

9. En el 4ABC, se sabe que los vertices B, C, el circuncentro O y el ortocentro H del


4ABC estan todos sobre una misma circunferencia. Figura 141.

a) Calcule el valor de A.

b) Demuestre que el incentro tambien pertenece al circuncrculo de BCOH.

Solucion:

a) Sea BAC = . Como O es el circuncentro del 4ABC, tenemos que BOC = 2. Por
otra parte, sabemos que al ser H ortocentro, se cumple que BHC = 180 . Ahora

101
bien, la condicion de que B, C, H y O son concclicos implica que BOC = BHC,
de donde 2 = 180 , y por tanto = 60 .

Figura 141

b) Este problema se basa en el siguiente resultado: si I es el incentro del 4ABC entonces


BIC = 90 + A 2
. Como en este caso A = 60, entonces BIC = 120 = BOC =
BHC, por lo que B, C, O, H, I, se ubican sobre una misma circunferencia.

10. Sea ABC un triangulo tal que las medianas respectivas a B y C son perpendiculares.
Demuestre que se cumple la relacion (Ver figura 142).

5BC 2 = CA2 + AB 2 .

Figura 142

Solucion: Sean BB 0 y CC 0 las medianas que son perpendiculares, y sea G el centroide.


Observe que el cuadrilatero BCB 0 C 0 tiene diagonales perpendiculares; por el teorema de
Pitagoras se cumple

BC 2 + B 0 C 02 = C 0 B 2 + B 0 C 2
 2  2  2
2 BC AB AC
BC + = +
2 2 2
2 2 2
5BC = AB + AC

102
11. Sea ABC un triangulo de ortocentro H. Sean P y Q los pies de las perpendiculares desde
H a las bisectrices interior y exterior de A, respectivamente. Si M es el punto medio de
BC, mostrar que P , Q y M estan alineados. Figura 143.

Solucion: Sean E y F los pies de las alturas trazadas desde a B y C, respectivamente.


Se sabe que AP AQ, por lo que AP HQ es un rectangulo. Como AP H = AQH =
AEH = AF H = 90, los puntos P , Q, E, F , pertenecen a una circunferencia de
diametro AH. Ademas, en esta circunferencia, como AP y AQ son bisectrices (interior
y exterior, respectivamente) del EAF , P y Q son los puntos medios del arcos EF ,
por lo que P Q es la mediatriz de EF . Por otra parte, como BEC = BF C = 90, el
cuadrilatero BCEF es cclico, y el circuncentro es M , por lo que M E = M F ; entones M
esta en la mediatriz de EF , la cual es P Q.

Figura 143

12. En un triangulo ABC, sea M el punto medio de BC. Si se cumple que AB 6= AC y


ademas M AC + ABC = 90 , hallar BAC. Figura 144.

Solucion: Sin perdida de generalidad, suponga que AB > AC. Sea N la interseccion de
AB con la mediatriz de BC. Se forma el 4BCN que es isosceles, entonces CN M =
90 M CN = 90 M BN = CAN , lo cual implica que el cuadrilatero ACM N es
cclico. Por lo tanto, BAC = BM N = 90.

13. Sea ABC un triangulo y U un punto de su circuncrculo tal que AU es bisectriz. Las
mediatrices en AB y AC cortan a AU en X y Y . Sea T la interseccion de BX con CY .
Demostrar que AU = T B + T C. Figura 145.
Solucion:41 Como X y Y pertenecen a las mediatrices de AB y AC, respectivamente,
y a la bisectriz AU , entonces 4ABX y 4ACY cumplen ser isosceles y semejantes entre
si, porque XBA = XAB = Y AC = Y CA = . Esto implica T XY = XBA +
41
El caso cuando AB = AC es trivial, porque X, Y y T colapsan en el circuncentro del 4ABC.

103
Figura 144

XAB = 2 = Y AC+Y CA = T Y X, es decir, el 4T XY es isosceles con T X = T Y .


Por otra parte, como ABU C es cclico, U BC = U AC = U AB = U CB = . De
aqu se concluye que 4U BC es isosceles, con U B = U C. Ademas, XU B = ACB =
Y CU y XBU = ABC = Y U C; por criterio ALA, 4U XB 4CY U , por lo que
BX = Y U . Finalmente, T B + T C = (BX T X) + (CY + T Y ) = Y U + AY = AU .

Figura 145

14. (The 59th Romanian Mathematical Olympiad Final Round) Sea ABCD un rectangulo de
centro O con AB 6= BC. La perpendicular en O a BD corta a las lneas AB y BC en los
puntos E y F , respectivamente. Sean M y N los puntos medios de los segmentos CD y
DA, respectivamente. Probar que las lneas rectas F M EN .

Solucion: Considere la figura 146, sin perdida de generalidad, se ha supuesto AB < BC.42
Sea L el punto medio de AB, y H es la interseccion de EF con AD. Se tiene que
LN k BD, y como BD EF entonces LN EF ; ademas, como ABCD es un rectangu-
lo, DA AB, por lo tanto, H es el ortocentro del 4ELN , y as, LH EN . Por otra
parte, las reflexiones de L y H con respecto a O son respectivamente M y F , por lo que
LH k M F , lo cual implica que F M EN .
42
El otro caso es completamente analogo.

104
Figura 146

15. Sea ABC un triangulo rectangulo, con A = 90 . Sea D un punto en su interior tal que
DAC = DCA = DBC = , y AC = BD. Determine el valor de . Figura 147.

Solucion: Sean P y Q los pies de las perpendiculares trazadas desde D hacia CA y AB,
respectivamente, R es un punto sobre BC tal que DB DR, y E es la interseccion de CD
con AB. Como el 4ACD es isosceles, P es punto medio de AC, entonces AC = 2P A =
2DQ = BD, por lo que el 4BDQ es un triangulo notable y DBQ = 30. Por otra parte,
por criterio ALA, 4ACE 4DBR, por lo que CE = BR; como P D k AE, D es punto
medio de CE; as, si M es el punto medio de BR (y circuncentro del 4BDR) se cumple
que DC = RM = DM , por lo que el 4CDM es isosceles. Por la relacion entre el angulo
inscrito y el angulo central DM R = 2DBR, por lo tanto DCR = 2. Sumando los
angulos internos del 4ABC se tiene A + B + C = 90 + 30 + + 3 = 180, lo cual
implica = 15.

Figura 147

16. Sea ABC un triangulo y M un punto tal que M AB = 10, M BA = 20, M AC = 40

105
y M CA = 30. Probar que el 4ABC es isosceles. Figura 148.
Solucion: Sea D la reflexion del punto A con respecto a la recta BM . Entonces el 4AM D
es isosceles con AM D = 2 (M AB + ABM ) = 60 y por lo tanto es equilatero. Tam-
bien DBA = 2M BA = 40 y como BAC = 50, implica que DB AC. Sea E la
interseccion de BD con CM , se cumple que CED = 90 ACE = 60 = M AD,
por lo que el cuadrilatero AM ED es cclico. De aqu, DEA = DM A = 60. Como
DEC = DEA y ED AC, se tiene que ED es bisectriz y altura en el 4AEC, por
lo tanto ED es mediatriz de AC, lo cual implica que BA = BC.

Figura 148

17. Teorema de Poncelet: Demuestre si 4ABC es un triangulo rectangulo con A = 90,


entonces 2(r + R) = b + c.

Figura 149: Teorema de Poncelet

Solucion: Sean O e I el circuncentro y el incentro del 4ABC. Como A = 90, O es el


punto medio de BC, por lo que a = 2R. Por otra parte, si P y Q son las proyecciones de I
sobre AB y AC, claramente AP IQ es rectangulo, pero como I es incentro IP = r = IQ,
por lo que AP IQ es cuadrado. Se sabe que para un triangulo cualquiera AP = s a, por
lo tanto
r = sa
b+ca
r =
2
2r + a = b + c
2(r + R) = b + c

106
18. En la figura 150, ABCD y P QRS son cuadrados, 4ABP 4BCQ 4CDR 4DAS
y los los radios de las cinco circunferencias son iguales a r. Si a es el lado del cuadrado
ABCD, determine r en funcion de a.

Figura 150

Solucion: Se tiene AB = a y se definen b = AP y c = BP ; observe que por las congruen-


cias BQ = b, por lo que P Q = cb = 2r. Por otra parte (analogamente a la demostracion
del teorema de Poncelet), al calcular el inradio del 4ABP se tiene que 2r = b + c a,
entonces c b = b + c a, lo cual implica que a = 2b. Por lo tanto, el 4ABP es un
triangulo notable de 30, 60, 90, y as
cb
r =

2
3
a 12 a
= 2
2
!
31
= a
4

107
19. Recta de Euler. El centroide G, el ortocentro H y el circuncentro O de un triangulo
estan alineados, y ademas GH = 2GO.

Figura 151: Recta de Euler

Solucion: Considere la siguiente figura. Sean AHa y BHb alturas, OA0 y OB 0 mediatrices.
Observe que HA BC y OA0 BC, por lo que HA k OA0 ; analogamente HB k OB 0 ;
tambien, por el teorema de la base media AB k A0 B 0 y AB = 2A0 B 0 . Esto implica que
4ABH ' A0 B 0 O y la razon de semejanza es 2; en particular AH = 2A0 O. Si definimos
G como la interseccion de la mediana AA0 con HO, claramente 4AHG ' 4A0 OG y la
razon de semejanza es la misma que la anterior, por lo que GA = 2GA0 , i.e., G es el
centroide del 4ABC. Esto implica que el ortocentro, el centroide y el circuncentro de un
triangulo estan alineados, y por la semejanza GH = 2GO.

108

You might also like